Xizmat sizning tilingizda ham mavjud. Tarjima qilish uchun bosingO'zbek
Best analytics service

Add your telegram channel for

  • get advanced analytics
  • get more advertisers
  • find out the gender of subscriber
Category
Channel location and language

all posts Юрист Хизмати | Расмий канал

Каналимиз № 0005006503 - сонли давлат рўйхатидан ўтган гувоҳнома асосида фаолият юритади. Саволларингизни каналимиз постларида кўрсатилган мурожаат ботларимиз орқали юборишингиз мумкин. Реклама бўйича: t.me/Yurist_Xizmati_reklama 
Show more
135 684-52
~39 640
~68
74.64%
Telegram general rating
Globally
19 498place
of 78 777
120place
of 430
In category
64place
of 313
Posts archive
2018 йилда сотилган уйдан солиқ тўлайманми? 4502-савол: 2015-йили бир одамнинг номига Андижон шахрининг иккита жойидан Уй олинади, Уша уйлар 2018- сентябрь ойида сотиб юборилди. 2024-апрель ойида Уша одамнинг номига даромад солиги келди. Шу келган даромад солиги тугрими, шу нарсани батафсил тушинтириб беринг.Шу хакида бирорта Карор борми? ЮРИСТ ЖАВОБИ: Маълумки 2019-йилнинг 30-декабрида янги таҳрирдаги Солиқ кодекси қабул қилинди. Сизнинг ҳолатингизда солиқ қарздорлиги эски таҳрирдаги Солиқ кодекси амалда бўлган. Лекин мулкий даромад бўйича иккала кодексда ҳам бир хил қоида кетган. Янги кодекснинг 375-моддасига кўра солиқ солинадиган мулкий даромадлар қаторига солиқ тўловчига мулк ҳуқуқи асосида тегишли бўлган мол-мулкни реализация қилишдан олинган даромадлар киради. Мол-мулкни реализация қилишдан олинган даромадлар мазкур мол-мулкни реализация қилиш суммасининг ҳужжатлар билан тасдиқланган уни олиш қийматидан ошган қисми сифатида аниқланади. (Бу қоида экси таҳрирдаги кодексининг 176-моддасида келтирилган.). ✅ Шундай экан бу борада қарздорлик бўйича шубҳа йўқ. Солиқ солинадиган даромад уй-жойнинг сотиб олиш ва сотиб юбориш ўртасидаги харажатлардан ташқари бўлган даромаддан ҳисобланади. ➡️ Энди ушбу солиқни ундириш масаласига келсак. Бу борада тўлиқ янги таҳрирдаги Солиқ кодексига юзланишимиз мумкин. Кодекснинг 88-моддасига биноан солиқ органи ёки бошқа ваколатли орган солиқ текширувини ўтказишга, текширув натижалари бўйича солиқ тўловчига солиқ қарзини узиш тўғрисида талабнома юборишга ёки солиқ тўғрисидаги қонунчиликка мувофиқ тўланиши лозим бўлган солиқлар миқдорини қайта кўриб чиқишга ҳақли бўлган муддат солиқ мажбурияти бўйича даъво қилиш муддати ҳисобланади. Агар ушбу Кодексда бошқача қоида белгиланмаган бўлса, солиқ мажбурияти бўйича даъво қилиш муддати, натижаларига кўра солиқ мажбурияти аниқланадиган солиқ даври тугаганидан кейин уч йилни ташкил этади. Агар солиқ мажбуриятининг юзага келиши муайян воқеа ёки ҳаракат билан боғлиқ бўлса, агар ушбу Кодексда бошқача қоида белгиланмаган бўлса, солиқ мажбурияти бўйича даъво қилиш муддати шу воқеа ёки ҳаракат содир бўлган пайтдан эътиборан уч йилни ташкил этади. ❗️Агар сизларнинг ҳолатингиз бўйича уйлар нотариал тасдиқланган олди-сотди шартномаси бўйича 2018-йил сотилган бўлса, у ҳолда солиқни ундириш бўйича даъво муддати ўтиб кетган ҳисобланади. Шундай экан бу қарздорлик ундирувга қаратилмаслиги керак. Каналимизга уланинг                 👇👇 👉 ‌‌
Show more ...
Юрист Хизмати Қабул
Ўзингизни қийнаётган барча саҳаларга оид муаммоли саволларингизни юборишингиз мумкин.
16 099
35
#Ана_холос Охиригача ўқинг: Машина олдим деб хурсанд бўлган экан, аслида... 500 $ га 2 ой вариантга Тико олдим, ўзимни номимга ўтказиш шарти билан Идорага бордик, тўлови 5 млн экан, акалар, мен адашяпманми ёки улар нима дейсизлар? 500$ га машина олдим, номимга ўтказиш 400$ бўлди. Маҳаллага миниб келсам, махалладагилар қуллуқ бўлсинга битта чойхонага туширишди. Вариантга олган машинамга чойхона қилдик, дастурхони билан 100 $ кетди. Ҳаммасини ҳазм қилдим, лекин янги ойда болалар пули чиқмагани алам қилди, сабаби номимда машина бормиш... Машина олмоқчи бўлган танишларга юборамиз!
Show more ...
23 668
287
#Биласизми ❗️25 йиллик иш стажига эга ўқитувчиларнинг тоифаси сақланади Маълумки, Мактабгача, умумий ўрта, ўрта махсус, профессионал ва мактабдан ташқари таълим ташкилотлари педагог кадрларини аттестациядан ўтказиш тартиби тўғрисидаги низом 4-бандига кўра мажбурий аттестация 5 йилда . Ушбу низом кучга киргандан сўнг бир қатор педагог ходимлар аттестатсияга жалб этилмаган ҳолда тоифаси сақланишига оид қатор имтиёзлар белгиланган. Хусусан, низом 6-бандига асосан: • педагогик фаолият бўйича 25 йил ва ундан кўп иш стажига эга бўлган педагог ходимлар; • ёшга доир пенсияга чиқиши учун 5 йилдан кўп бўлмаган вақт қолган педагог ходимлар навбатдаги мажбурий аттестацияга жалб этилмайди. Бунда 8-банди Вазирлар Маҳкамасининг 2023 йил 29 декабрьдаги 692-сонли қарорига асосан «е» кичик банд билан тўлдирилган, унга кўра педагогик фаолият бўйича 25 йил ва ундан кўп иш стажига эга бўлган ҳамда ёшга доир пенсияга чиқиши учун 5 йилдан кўп бўлмаган вақт қолган педагог кадрларга — уларнинг амалдаги малака тоифаси (лавозими) берилади. Шунга кўра 25 йил ёки ундан кўп иш стажига эга педагогларнинг ҳамда пенсия ёшига чиқишига 5 йил ва ундан кам вақт қолганда педагогларнинг тоифаси сақланади. Уларга берилган сертификатнинг амал қилиш муддати тугайдиган вақтда низомнинг 32-бандига мувофиқ педагоглар ҳар йили 1 январьдан 31 декабрга қадар тўғридан-тўғри аттестациядан ўтказилмасдан малака тоифаси (лавозим) сертификатини олиш учун бевосита давлат хизматлари марказлари орқали ёки Ягона интерактив давлат хизматлари портали ёхуд ваколатли вазирликларнинг расмий веб-сайтида рўйхатдан ўтган ҳолда мурожаат қилишлари лозим. Бунда педагогларга тўғридан-тўғри аттестациядан ўтказилмасдан малака тоифаси (лавозим) сертификатини олиш бўйича сўровномаси 20 кун муддатда кўриб чиқилади ва ижобий қарор қабул қилинганда уларга ҚР-код қўйилган электрон шаклдаги сертификат берилади. Батафсил: Каналимизга уланинг                 👇👇 👉 ‌‌
Show more ...
Юрист Хизмати Қабул
Ўзингизни қийнаётган барча саҳаларга оид муаммоли саволларингизни юборишингиз мумкин.
36 773
862
ЎИБДЎ ҳар ой ўқитувчилардан ойлик ҳисоботларни сўраши мумкинми? 4501-савол: Assolomu aleykum savolim bor edi. Man maktabda o‘qituvchi bo‘lib ishlayman savolim shuki maktabda maktab rahbariyatiga yani zauchga har oyda o‘qituvchilar ped yuklama bo‘icha qanday hujjat topshirishlari kerak. Maktab zauchi maktabda har oyda o‘qituvchilardan ped yuklama bo‘icha hujjat talab qiladi. O‘qituvchilar qonun bo‘icha maktab zauchiga har oyda ped yuklama bo‘icha hujjat topshirishlari kerakmi. Zauch ped yuklama bo‘icha o‘qituvchilardan hujjat talab qilishi qonunan to‘grimi ЮРИСТ ЖАВОБИ: Мактабгача ва мактаб таълими вазирининг 2023-йил 20 - декабрдаги 565-сонли буйруғига 37-иловада тасдиқланган Умумий ўрта таълим муассасалари (айрим фанлар чуқур ўрганиладиган ихтисослаштирилган муассасалар) ходимларининг лавозим мажбуриятларининг ўқув ишлари бўйича директор ўринбосарининг лавозим мажбуриятлари қаторида қуйидагилар: 👉 педагогик ходимга белгиланган иш юкламасини, жумладан, ўқув жараёни, ўқув жараёнининг методик таъминоти ва ташкилий-педагогик жараёнларга оид ишларни ўз вақтида бажарилишини назорат қилиш ва таъминлаш; 👉 ўқитувчилар фаолиятининг йиллик, чораклик ва ойлик режаларини тасдиқлаш ва бажарилишини назорат қилиш, ўқитувчилар фаолиятининг тасдиқланган режаларига мос равишда иш вақти ҳисобини олиб бориш белгиланган. ❗️Мана шу қоидага кўра сизлардаги ҳолатда ЎИБДЎ сизларда ойлик ҳисоботларни сўраши ҳуқуқий асосга эга. Каналимизга уланинг                 👇👇 👉 ‌‌
Show more ...
Юрист Хизмати Қабул
Ўзингизни қийнаётган барча саҳаларга оид муаммоли саволларингизни юборишингиз мумкин.
39 663
653
⚡️Энди электр энергияни тежаб ишлатмасак бўлмайди Юқоридаги жадвалдан ишлатадиган электр энергиянгиз сарфини тахминий ҳисоблаб кўринг 👉
44 134
941
❗️Ходимнинг аризасини мажбуран олиб, уни ишдан бўшатиш ноқонуний 4500-савол: Assalomu alaykum. Men 2018- yildan buyon maktabda MMIBDO‘ bosari lavozimida ish olib boraman. Prezidentimiz Shavkat Miromovich tashabbusi bilan davomatni yo‘lga qo‘yish asosida Milliy givardiya xodimlari bilan birgalikda "Xavfsiz maktab" konsepsiyasini keng joriy etish,"Oila - mahalla-maktab" uzviy tizimi asosida voyaga etmaganlar bilan manzilli ishlashni yo‘lga qo‘yish  maqsadida 2024 yil 26 fevralda Maktabgacha va maktab ta‘limi vazirligi hamda Milliy givardiya o‘rtasida 4-k/k va 5-k/k sonli qo‘shma qarori imzolangan. Bu qarorga asosan 1336 ta o‘quvchidan 6 ta o‘quvchi darsga kelmagan. 4-sababli va 2-sababsiz ma‘lumot berganligim uchun M va MTB mudiri va maktab direktori bilan birgalikda majburlab ishdan bo‘shash haqida ariza yozdirib oldi. Ammo arizamni yarim soatda qaytarib oldim. Lekin M va MTB mudiri seni arizang telifonimda rasmi bor deb ta‘kidlayabdi. Menga shu masalada yordam berishingizni so‘rayan. ЮРИСТ ЖАВОБИ: 🙅‍♂ Ходимнинг аризасини мажбуран олиб, уни ишдан бўшатиш ноқонуний. Меҳнат кодексининг 160-моддасига асосан ходим номуайян муддатга тузилган меҳнат шартномасини, шунингдек муддати тугагунига қадар муддатли меҳнат шартномасини ўн тўрт календарь кун олдин бу ҳақда иш берувчини ёзма шаклда огоҳлантирган ҳолда бекор қилишга ҳақли. Бунда ходим ўзи берган аризани чақириб олишга ҳақлидир. ❗️Ходим аризани ўз ташаббуси билан (ўз хоҳиши билан) ёзади. Агар иш берувчи аризани бошқа ҳар қандай усулда олса, унда ходимда ишга тикланиш ҳуқуқи пайдо бўлади. Қаранг! Олий суди пленумининг 20.11.2023 йилдаги 26-сон “Судлар томонидан меҳнат шартномасини бекор қилишни тартибга солувчи қонунчиликни қўллаш амалиёти тўғрисида”ги қарори 11-бандига биноан меҳнат шартномасини бекор қилишга фақат ходимнинг ёзма аризаси асос бўлиб, у ҳақиқатдан унинг меҳнат муносабатларини бекор қилиш тўғрисидаги хоҳиш-истагини намоён этиши лозим. Бунда қарор 20-бандига кўра қонунда белгиланган ёки ходим ва иш берувчи келишувига кўра қисқартирилган огоҳлантириш муддати давомида ходим меҳнат шартномаси бекор қилинганлиги ҳақида буйруқ чиқарилган бўлишидан қатъий назар аризасини қайтариб олишга ҳақли. ✅ Шундай экан сизда аризани қайтариб олиш ҳуқуқи мавжуд. ❗️Ишдан бўшатиш ҳақида буйруқ чиқаришда эса иш берувчи телефондаги расмни эмас, аксинча аризанинг асл нусхасини олиши керак. Меҳнат шартномасини бекор қилишга ходим шу ташкилотдаги ишни ҳақиқатда ташлаб кетиш истагини билдирган ёзма аризасига биноан йўл қўйилади (Қарор 21-банди). Бунда судларнинг эътибори, ходимни ариза ёзишга нафақат мажбурлаш, босим ёки тазйиқ ўтказиш, балки ходимда меҳнат муносабатларини тугатиш истаги бўлмай туриб, иш берувчи томонидан уни ариза ёзишга фаол равишда ундаш ҳаракатлари ҳам қистов сифатида баҳоланиши лозимлигига қаратилган. ✅ Шунга кўра сиз бу вазиятда туман адлия бўлимига мурожаат қилишингиз лозим. Шуни ҳам айтиш керакки, мактабларда ўринбосарларини ишдан бўшатиш фақат иш берувчининг ваколатига тегишлидир. Ўзбекистон Республикаси Президентининг 26.05.2023 йилдаги ПФ-79-сон Фармонига кўра умумий ўрта таълим муассасалари директорларига ўз ўринбосарларини ишга қабул қилиш ва ишдан бўшатиш ҳуқуқи берилган. Яъни иш берувчи директор ҳисобланади. ❌ Шундай экан сизни ММТБ мудири ишдан бўшатиши яна бир асосга кўра ноқонунийдир. Каналимизга уланинг                 👇👇 👉 ‌‌
Show more ...
Юрист Хизмати Қабул
Ўзингизни қийнаётган барча саҳаларга оид муаммоли саволларингизни юборишингиз мумкин.
28 294
255
Интернет ишламай қолгани учун қайта аттестациядан ўтиш мумкинми? 4499-савол: Ассалаумалейкум, мен Қорақалпоғистон Республикаси Қунғирот туманига қарашли 54 мактабнинг қозоқ синфининг бошлангич синф укитувчисиман  19-апрель куни соат10:30да атестация тест синовига катинашдим Синов пайтида интернет алокаси учиб колди, 37дакикани курсатиб компьютерим котиб колди Интернет тикланган пайтида озгина дакика колди деган куркув булиб, саволларни чала укиб, нотугри белгиладим Бир тарафдан аудиторияда интернет ишламай колган пайтида дакикалари хисобланиб кетган укитувчилар шовкин чикаришди, тестни тугри мулохоза килишимга шовкин хам халкит юерди Тестлар козок тилига нотугри таржима килинган, купчилик тест мактабимизга хали етиб келмаган янги китоблардан олинган 19-апрель кунги топширилган тест синови буйича аппеляция ёзиб кетдим, лекин бу курилади ми йук ми ишончим комил эмас Бу холатда яна каерга мурожат килишим мумкин? Айнан 19-апрель куни 10:30да топширган узтозларнинг ахволларига юрагим ачиди Бу холатда тестни кайта топширишга хакким борми? ЮРИСТ ЖАВОБИ: ❗️Бу ҳолатда зудлик билан аттестация натижаси устидан апелляция беришингиз лозим. Вазирлар Маҳкамасининг 2021 йил 17 сентябрдаги 572-сон қарорига 1-илова билан тасдиқланган “Мактабгача, умумий ўрта, ўрта махсус, профессионал ва мактабдан ташқари таълим ташкилотлари педагог кадрларини аттестациядан ўтказиш тартиби тўғрисида”ги низом 55-бандига кўра аттестация натижаларидан норози бўлган педагог кадрлар натижалар эълон қилинган кундан бошлаб ўн кун мобайнида апелляция комиссиясига ёзма равишда ариза (шикоят) беришлари мумкин. Бунда ариза (шикоят) рўйхатга олинган кундан бошлаб ўн кун муддатда кўриб чиқилади. Низомнинг 59-бандига кўра апелляция комиссияси педагог кадрнинг аттестация жараёнларига оид мурожаатини кўриб чиқиб, қуйидаги қарорлардан бирини қабул қилади: 👉 педагог кадрнинг тўғри баҳоланганлиги, ўқув фани бўйича тест ва малака синови материалларида хато ва камчиликлар, шунингдек, синовларни ўтказиш жараёнида ташкилий-техник жиҳатдан камчиликлар аниқланмаган тақдирда, аттестация натижаларини ўзгартиришсиз қолдириш; 👉 педагог кадрнинг педагогик маҳорати ва психологик тайёргарлиги нотўғри баҳоланганлиги, ўқув фани бўйича тест ёки малака синови материалларида хато ва камчиликлар мавжудлиги аниқланган тақдирда, аттестация натижаларини ўзгартириш; 👉 ўқув фани бўйича тест ёки малака синовларини ўтказиш жараёнида ташкилий-техник жиҳатдан камчиликлар мавжудлиги аниқланган тақдирда, тест синовларининг қайтадан топширилишини ташкил этиш. ✅ Шунга кўра сиздаги ҳолатда техникум муаммолар аниқлаган тақдирда сизлар қайтадан аттестация синовларидан ўтишингиз мумкин бўлади. Каналимизга уланинг                 👇👇 👉 ‌‌
Show more ...
Юрист Хизмати Қабул
Ўзингизни қийнаётган барча саҳаларга оид муаммоли саволларингизни юборишингиз мумкин.
28 407
142
Телеграм бот орқали фирибгарларга пулимни олдириб қўйдим, энди нима қилсам бўлади? 4498-савол: Ассалому алайкум телеграм бот орқали кибер товламачига пулимни олдириб қўйдим бирор ёрдам бера оласизми? ЮРИСТ ЖАВОБИ: ❗️Ҳурматли фуқаро бу вазиятда зудлик билан туман прокуратурасига ариза беришингиз лозим. Бунда ўша шахсларга нисбатан Жиноят кодексининг 168-моддаси билан жиноят иши очилади. ❕Лекин бир нарсани эътиборга олиш керак. Прокуратура ёки бошқа органлар ўша шахсларни аниқлай олсагина жавобгарлик чораси қўлланилади. Каналимизга уланинг                 👇👇 👉 ‌‌
Show more ...
Юрист Хизмати Қабул
Ўзингизни қийнаётган барча саҳаларга оид муаммоли саволларингизни юборишингиз мумкин.
27 187
55
⚡️1 июлдан йўл бўйидаги мактаблар олдида махсус радарлар ўрнатилади Шунингдек, фото ва видео қайд этиш қурилмалари: ▪️камида тўрт тасмали йўллардаги пиёдалар ўтиш жойларида, ▪️кўп авария бўладиган жойларда, ▪️темир йўл кесишмаларида босқичма-босқич .

QAROR 148 SON TG (online-video-cutter.com).mp4

38 684
254
Дарёдан шағал қазиб олиш тақиқландими? 4497-савол: Assalom alaykum men yuk mashinasida ishtixon tumanidagi daryodan shaģal tosh yuklab chiqayotganimda meni pastdarģom ekologiya hodimlari ushlab jarima qulashi haqida ogohlantirishdi pastdarģom ekologiya hodimlari ishtixon tuman hudidda shtraf yozish huquqi bormi. ЮРИСТ ЖАВОБИ: Маъмурий жавобгарлик тўғрисидаги кодекс 197-6-моддасига асосан экология ва атроф-муҳитни муҳофаза қилиш органлари давлат инспекторининг қонуний фаолиятига тўсқинлик қилганлик, шу жумладан унинг текшириш ўтказишига тўсқинликларни юзага келтирганлик, хўжалик юритувчи субектларнинг ҳудудлари ва иншоотларини кўздан кечириш учун уларга киритишни рад этганлик, шунингдек экология ва атроф-муҳитни муҳофаза қилиш тўғрисидаги қонунчиликнинг бузилишларини бартараф этишга доир кўрсатмаларини бажармаслик, ➖ фуқароларга базавий ҳисоблаш миқдорининг бир бараваридан уч бараваригача, мансабдор шахсларга эса — 3 бараваридан 5 бараваригача миқдорда жарима солишга сабаб бўлади. ☝️Мана шу қоидага кўра экология органи ходимининг ҳаракатлари ҳуқуқий асосга эга. 💁‍♂ Шуни ҳам айтиш керакки, Ўзбекистон Республикаси Президентининг 17.01.2024 йилдаги ПФ-14-сон “Сув ҳавзаларида норуда материалларни қазиб олишни тартибга солиш чора-тадбирлари тўғрисида”ги фармонига асосан 2024-йил 1-майдан бошлаб Чирчиқ, Сирдарё, Сангзор, Зарафшон, Норин, Қашқадарё ва Сурхондарё дарёлари ўзанларидаги норуда материалларни қазиб олишга муддатсиз мораторий жорий қилинди. ❌ Шунга кўра 1-майдан бошлаб дарёлардан қум-шағал қазиб олиш тақиқланади. ❗️Бунда мораторий даврида дарё ўзанлари, соҳил бўйи минтақалари ва сувни муҳофаза қилиш зоналаридан қум-шағал материалларини ноқонуний қазиб олиш натижасида табиатга етказилган зарар учун ундириш суммалари 10 бараварга оширилган ҳолда қўлланилади Каналимизга уланинг                 👇👇 👉 ‌‌
Show more ...
Юрист Хизмати Қабул
Ўзингизни қийнаётган барча саҳаларга оид муаммоли саволларингизни юборишингиз мумкин.
32 847
131
Сайёҳлик агентлиги очмоқчиман, мактаб ўқувчиларини экскурсияга олиб юриш тартиби қандай? 4496-савол: Ассалому алейкум! Мен сайёхлик агентлиги очмокчиман. Фаолиятимиз мактаб укувчиларни маданий хордик жойларга олиб бориш( театирга, вилоятларга, заводларга). Биз купинча автобусда мактаб укувчиларни олиб юрамиз. Мактаб укувчиларни олиб юриш тартиб ва конунлари кандай(халк таълимига ёзиш, гаи ходимларидан рухсат, автобус техник курик)? ЮРИСТ ЖАВОБИ: Вазирлар Маҳкамасининг 2003-йил 4-ноябрдаги 482-сон қарорига 1-иловада тасдиқланган Автомобил транспортида йўловчилар ва багажни ташиш Қоидалари 93-бандига кўра болаларни ташишни ташкил этишга буюртманомалар ташувчилар томонидан гуруҳларни ўқитувчиларнинг ёки махсус белгиланган катта ёшдаги кишиларнинг (битта катта ёшдаги кишига 15 боладан кўп бўлмаган) кузатиб бориши шартлари бажарилганда қабул қилинади. ❗️Бунда мактаб ёшидаги болалар гуруҳларини ташишда уларга ҳамроҳлик қилиш учун тиббиёт ходимлари ажратилади. Демак, сиздаги вазиятда ҳам болаларни ташишда мазкур тартибда риоя қилиш лозим. Энди қаранг! Юқоридаги қарорга 2-иловада тасдиқланган Ўзбекистон Республикасида автобусларда йўловчилар ташиш хавфсизлигини таъминлашга доир Талаблари 40-бандига кўра таълим муассасаси маъмурияти (ташкилотчи) ташувчи билан шартнома тузилганидан кейин жўнаб кетиш санасигача уч иш кунидан кечикмай ҳаракатланиш санаси ва қатнов йўлини, шунингдек, барча йўловчилар рўйхатини, автобуслар тўғрисидаги (мансублилиги, маркаси, давлат рўйхатдан ўтказиш рақами белгиси) тўғрисидаги маълумотларни кўрсатган ҳолда ҳамроҳликни ташкил этиш учун буюртманомани Ўзбекистон Республикаси Ички ишлар вазирлигининг Йўл ҳаракати хавфсизлиги давлат хизматига киритади. ☝️Мана шу қоидага кўра сиз билан шартнома тузгандан кейин мактаб раҳбарияти юқоридаги ҳаракатни амалга ошириши керак бўлади. Бунда Ўзбекистон Республикаси Ички ишлар вазирлигининг Йўл ҳаракати хавфсизлиги давлат хизмати томонидан бориладиган қатнов йўлида об-ҳаво ёки йўл шароитларининг номувофиқлиги, шу жумладан, бориш пунктига ҳаракатланишда чекланишлар мавжудлиги, шунингдек, транспорт воситалари ва ҳайдовчилар малакасининг йўл ҳаракати хавфсизлиги талабларига номувофиқлиги ҳолатларида ҳамроҳлик қилиш рад этилиши ҳақида қарор қабул қилиниши мумкин. ✅ ДЙХХ ижобий хулоса берган тақдирда алоҳида кузатувчи ажратади. Шундан сўнг болаларни экскурсияга олиб бориш мумкин бўлади. Каналимизга уланинг                 👇👇 👉 ‌‌
Show more ...
Юрист Хизмати Қабул
Ўзингизни қийнаётган барча саҳаларга оид муаммоли саволларингизни юборишингиз мумкин.
35 866
164
Бугунги хабар бўйича тушунтириш берилди Ички ишлар вазири ўринбосари Бекмурод Абдуллаевнинг маълум қилишича, 48 соат ҳақидаги талаб фото ёки видеокамералар ўрнатган субйектларга тааллуқли. Яъни камералар орқали қайд этилган қоидабузарликлар 48 соат ичида жарима ҳақида қарор чиқарадиган ваколатли органга, яъни ЙПХ марказига юборилиши керак. Агар тасвирлар ЙПХга 48 соатдан кечикиб келса, ўша ҳолат бўйича иш очилмайди. ❓Хўш, унда ҳайдовчига қанчада етиб келиши керак? ЙПХ маркази маълумот келиб тушгандан кейин 15 кундан кечиктирмай жарима солиш ҳақида қарор чиқариши керак. Ундан кечикса, ўша қарор қонуний ҳисобланмайди. Хулоса: 2 кун (48 соат) — фото ёки видеокамералардан ЙПХ марказига келиб тушиш муддати; 15 кун — ваколатли органнинг қарор қабул қилиш муддати. Жами 17 кун. ✅ Демак, маъмурий жавобгарликка тортиш ҳақидаги қарор қоидабузарлик содир этилгандан сўнг 17 кундан кечикиб чиқарилса, ҳайдовчиларга жарима солинмайди.
Show more ...

file

45 107
487
Ўриндош ва соатбай ўқитувчиларга ҳам сертификат учун устама тўланадими? 4495-савол: Sertifikat uchun soatbay ya'ni oʻrindosh oʻqituvchilar uchun ham toʻlanishi haqidagi qaror kerak iltimos? ЮРИСТ ЖАВОБИ: Ўзбекистон Республикаси Президентининг 11.05.2022 йилдаги ПФ-134-сон Фармони 9-бандига кўра халқаро тан олинган сертификатга эга бўлган педагог кадрларга — уларнинг тариф ставкасига нисбатан 50 фоиз миқдорида ҳар ойлик қўшимча устама тўланиши белгиланган. Ушбу фармон ижросини таъминлаш мақсадида Мактабгача ва мактаб таълими вазирлиги ҳамда Иқтисодиёт ва молия вазирлигининг 2023 йил 28 июлда 25-сонли қўшма қарор ишлаб чиқилган ва бу қарор 1-августдан кучга кирди. Унга кўра С1 сертификат учун устама фақат фан ўқитувчиларига ўзлари дарс берадиган фан соатларидан келиб чиқиб тўланиши белгиланган. ✅ Ушбу қарорда устамалар ўриндошлик асосида ишлайдиганларга ҳамда соатбай дарс берадиган ўқитувчиларга ҳам тўланиши белгиланган. ❗️Шунга кўра сиздаги вазиятда соатбай асосдаги дарсларга ҳам устама тўланиши лозим деб ҳисоблаймиз. Бу вазиятда туман адлия бўлимига мурожаат қилиш лозим. Каналимизга уланинг                 👇👇 👉 ‌‌
Show more ...
Юрист Хизмати Қабул
Ўзингизни қийнаётган барча саҳаларга оид муаммоли саволларингизни юборишингиз мумкин.
34 085
265
❗️Фото ва видео-радар маълумотлари 48 соат ичида юборилмаган бўлса, ҳайдовчи жаримага тортилмайди Янги қонун лойиҳасида махсус автоматлаштирилган фото ва видео воситалари орқали қайд этилган йўл ҳаракати қоидалари бузилишига оид маълумотлар жарима солиш тўғрисида қарор чиқаришга ваколатли бўлган мансабдор шахсга 48 соат ичида юборилмаган бўлса ишни кўриб чиқишни тугатиш . Мажлисда қонун лойиҳаси депутатлар томонидан биринчи ўқишда қабул қилинди.
Фото ва видео-радар маълумотлари 48 соат ичида юборилмаган бўлса, ҳайдовчи жаримага тортилмайди
Янги қонун лойиҳасида махсус автоматлаштирилган фото ва видео воситалари орқали қайд этилган йўл ҳаракати қоидалари бузилишига оид маълумотлар жарима солиш тўғрисида қарор чиқаришга ваколатли бўлган мансабдор шахсга 48 соат ичида юборилмаган бўлса ишни кўриб чиқишни тугатиш белгиланмоқда.
37 308
304
⚡️2024 йил 1 майдан Ёлғиз кексалар ва ногиронлиги бўлган шахсларга ҳар ойда моддий ёрдам юерилади Қонунчиликка мувофиқ, реестрга киритилган ёлғиз кексалар ва ногиронлиги бўлган шахсларга ҳар ойлик уй-жой коммунал хизматлар ҳақини тўлаш бўйича компенсация ҳамда озиқ-овқат маҳсулотлари ва шахсий гигиена товарлари учун ҳар ойда қўшимча пул тўловлари ўрнига ёлғиз кексалар ва ногиронлиги бўлган шахслар учун ҳар ойда минимал истеъмол харажатлари миқдорида моддий ёрдам
Huquqiy axborot
#Қонунчиликдаги_янгиликлар ❗️2024 йил 1 апрелдан айрим ёлғиз кексаларга поездга бепул чипталар ажратилади “Ўзгалар парваришига муҳтож кексалар ва ногиронлиги бўлган шахсларга ижтимоий хизмат ва ёрдам кўрсатиш тизимини такомиллаштириш чора-тадбирлари тўғрисида”ги Президент қарори (ПҚ–410-сон, 27.12.2023 й.) қабул қилинди 📝 Қарорга кўра, “Инсон” ижтимоий хизматлар маркази томонидан ўзгалар парваришига муҳтож ёлғиз яшовчи ҳамда ёлғиз кексалар ва ногиронлиги бўлган шахсларнинг реестри юритилади. 🗓 2024 йил 1 апрелдан реестрга киритилган ёлғиз кексалар ва ногиронлиги бўлган шахсларга ва уларга бевосита ҳамроҳлик қилувчи 1 нафар шахсга бир йилда бир марта темир йўл ёки шаҳарлараро автобус учун бепул чипталар (бориш ва қайтиш учун) ажратилади. 🗒 2024 йил 1 майдан реестрга киритилган ёлғиз кексалар ва ногиронлиги бўлган шахсларга ҳар ойлик уй-жой коммунал хизматлар ҳақини тўлаш бўйича компенсация ҳамда озиқ-овқат маҳсулотларива шахсий гигиена товарлари учун ҳар ойда қўшимча пул тўловлари ўрнига ёлғиз кексалар ва…
41 834
603
Буйруқдаги санани хато киритгани учун кадрга интизомий жазо қўллаш мумкинми? 4494-савол: Assalomu alaykum men sentyabrdan buyon maktabda kadrlar bo'yicha menejer bo'lib ishlayman.Bir xodimga xomiladorlik tug'ruq ta'tiliga ketish haqida buyruq chiqargan edim. Ayrim texnik sabablarga ko'ra buyruqqa xodimning ta'tilga chiqish kuni xato ketib qolibdi lekin buyruqni qolgan qismlari to'g'ri. Yuridik markaz xodimlari shuni muvofiqlashtirib qaytadan yuborsangiz bo'ladi Qaytsadan yuborsam Yuridik Markaz xodimi buyruq chiqargan xodimga nisbatan intizomiy jazo qo'llash buyruq chiqaring debdi  Shu to'g'rimi? ЮРИСТ ЖАВОБИ: ❗️Интизомий таъсир чораси ходим ҳақиқатда ўз меҳнат мажбуриятларини бажармаганда қўлланилади. Меҳнат кодексининг 313-моддаси учинчи қисмида интизомий жазо чорасини қўллашда содир этилган қилмишнинг оғир-енгиллиги, унинг содир этилиши ҳолатлари, ходимнинг аввалги иши ва хулқ-атвори ҳисобга олиниши белгиланган. Мактабгача ва мактаб таълими вазирининг 2023-йил 20 - декабрдаги 565-сонли буйруғига 37-иловада тасдиқланган Умумий ўрта таълим муассасалари (айрим фанлар чуқур ўрганиладиган ихтисослаштирилган муассасалар) ходимларининг лавозим мажбуриятларига асосан Кадрлар бўйича инспектор (менежер)нинг асосий мажбурияти меҳнатга оид қонун ҳужжатларига мувофиқ ходимларнинг шахсий таркиби бўйича буйруқларни (ишга қабул қилиш, бошқа ишга ўтказиш, меҳнат шартномасини бекор қилиш, меҳнат таътили бериш ва ҳ.к.) расмийлаштириш ва уларнинг белгиланган тартибда сақланишини таъминлаш этиб белгиланган. ✅ Шунга кўра сизга бу мажбуриятни тўғри бажармаганингиз учун интизомий таъсир чораси қўллаш мумкин. Бунда Меҳнат кодексининг 312-моддасига мувофиқ меҳнат интизомини бузганлиги учун иш берувчи ходимга қуйидаги интизомий жазо чораларини қўллашга ҳақли: 1) ҳайфсан; 2) ўртача ойлик иш ҳақининг ўттиз фоизидан кўп бўлмаган миқдорда жарима. Ички меҳнат тартиби қоидаларида ходимга ўртача ойлик иш ҳақининг эллик фоизидан кўп бўлмаган миқдорда жарима солиниши ҳоллари назарда тутилиши мумкин. 3) меҳнат шартномасини бекор қилиш. Ушбу қоиданинг мазмунига кўра ходим меҳнат интизомини бузганда унга юқоридаги таъсир чораларидан бири қўлланилади. Бунда ходим содир этган қилмиш учун фақат битта интизомий таъсир чораси қўллаши мумкин. Ходимга интизомий таъсир чораси қўллаш тартиби МК 313-моддасида белгилган. Ушбу нормага мувофиқ иш берувчи ходимдан интизомий жазо чораси қўлланилгунига қадар ёзма тушунтиришни талаб қилиши, шу жумладан, агар интизомий қилмиш хизмат текшируви натижаларига кўра аниқланган бўлса, талаб қилиши шарт. Ходимнинг ёзма тушунтириш тақдим этишни рад этганлиги интизомий жазо чораси қўлланилиши учун монелик қилмайди ва ҳозир бўлган гувоҳлар кўрсатилган ҳолда далолатнома билан расмийлаштирилади. Шунга кўра сиздаги ҳолатда тушунтириш хати ёзишдан бош тортиш ҳолати бўлганда иш берувчи далолатнома тузган ҳолда интизомий таъсир чорасини қўллаши мумкин. Шуни ҳам айтиш керакки, интизомий жазо чорасини қўллашда содир этилган қилмишнинг оғир-енгиллиги, унинг содир этилиши ҳолатлари, ходимнинг аввалги иши ва хулқ-атвори ҳисобга олинади. 💁‍♂ Шу қоидага кўра иш берувчи сиздаги вазиятни мазкур мезонлар асосида баҳолаши, шундан сўнггина интизомий таъсир чораси қўллаши ёки қўлламаслиги мумкин. Каналимизга уланинг                 👇👇 👉 ‌‌
Show more ...
Юрист Хизмати Қабул
Ўзингизни қийнаётган барча саҳаларга оид муаммоли саволларингизни юборишингиз мумкин.
33 430
203
Аттестациядан ўтиш муддатини умра зиёрати сабабли кўчириш мумкинми? 4493-савол: Assalomu alaykum, o'qituvchiman. Ona tili va adabiyot fanidan attestatsiya sinovi 23.04dan 26.04gacha bo'lar ekan. Biz 16.04 kuni Umra safariga ketmoqchi edik. Ma'lumki, uzrli sababga ko'ra (sog'ligi tufayli) qatnasholmagan o'qituvchilar, ma'lum muddatdan so'ng sinovdan o'tishadi. Savolim: Umra safari ham uzrli sababga kiradimi. Men ham attestatsiya o'tgach, sinovga qatnasha olamanmi ЮРИСТ ЖАВОБИ: Олдиндан режалаштирилган (аттестация муддати белгиланмасдан олдин фирма билан шартнома тузилган бўлса) ҳамда билетлар аттестация муддати аниқланмасдан олдин олинган бўлса, умра зиёрати узрли сабаб деб топилиши мумкин. Вазирлар Маҳкамасининг 2021 йил 17 сентабрдаги 572-сон қарорига 1-илова билан тасдиқланган “Мактабгача, умумий ўрта, ўрта махсус, профессионал ва мактабдан ташқари таълим ташкилотлари педагог кадрларини аттестациядан ўтказиш тартиби тўғрисида”ги низом 45-бандига кўра педагог кадрлар аттестацияда узрли сабабга (вақтинча меҳнатга қобилиятсизлиги, давлат ёки жамоат вазифаларини бажарганлиги, республика ва халқаро мусобақаларда иштирок этганлиги, вақтинча хорижий мамлакатда сафарда бўлганлиги, яқин қариндошининг вафоти, ваколатли давлат органи ёки таълим ташкилотлари раҳбарларининг маълумотномасида кўрсатилган бошқа асосларга) кўра келмаган бўлса, уларнинг аттестацияси бошқа муддатга кўчирилади. ☝️Эътибор берган бўлсангиз, ушбу сабаблар ичида “вақтинча хорижий мамлакатда сафарда бўлганлиги” сабаби ҳам келтирилган. ✅ Шунга кўра сиздаги вазиятда аттестациядан ўтиш муддатини бошқа санага кўчириш мумкин деб ҳисоблаймиз. Каналимизга уланинг                 👇👇 👉 ‌‌
Show more ...
Юрист Хизмати Қабул
Ўзингизни қийнаётган барча саҳаларга оид муаммоли саволларингизни юборишингиз мумкин.
33 540
60
#Биласизми ❗️2024 йил 1 май кунидан аҳоли учун белгиланган электр энергияси ва табиий газ тарифлари Батафсил инфографикада Каналга уланиш👇👇👇
58 030
1 273
Таянч докторантурада академик таътил олиш мумкинми? 4492-савол: Assalomu aleykum men 1-kurs tayanch doktoranturada uzoq viloyatda o'qiyman. Sentyabr oyidan, ya'ni birinchi kursni tugatmasdan akademik ta'til olsam bo'ladimi? Shunda bir yil o'tib yana birinchi kursning boshidan o'qiymanmi? ЮРИСТ ЖАВОБИ:Таянч докторантурада академик таътил олиб бўлмайди. Лекин Вазирлар Маҳкамасининг 2017-йил 22-майдаги 304-сон қарорига 2-Илова Олий таълимдан кейинги таълим тўғрисидаги Низом 32-бандига кўра тегишли ҳужжатлар мавжуд бўлган тақдирда стажор-тадқиқотчилик, таянч докторантура ва докторантурада ўқиш муддати узоқ вақт касал бўлиш, ҳомиладорлик ва туғиш таътилида бўлиш даврига, шунингдек ҳарбий йиғинларга чақирилиш муносабати билан узайтирилиши мумкин. ✅ Шунга кўра сиздаги вазиятда ҳам ушбу ҳолатлардан бири мавжуд ва уни тасдиқлайдиган маълумотлар бўлганда ўқиш муддати узайтирилиши мумкин. Каналимизга уланинг                 👇👇 👉 ‌‌
Show more ...
Юрист Хизмати Қабул
Ўзингизни қийнаётган барча саҳаларга оид муаммоли саволларингизни юборишингиз мумкин.
34 380
31
Қўшним уйимга сув ўтказмаяпти, нима қилсам бўлади? 4491-савол: Ассалому алайкум бизга 1990 йилда асосан 6 сотхдан ер берилган бизда хозир сув муаммоси булябти биринчи ва иккинчи кушним уртасидан сув утадиган ташлама арикбор еди бу арикча юкори махалла кушниларни уйларини уртасидан утади биринчи Кушним шу арикни беркитиб 2017йилда кадастр хужжат килдирган екан. Бизга хозир сув бермаябди. Шу муаммони ечими топилмаябти улар килган кадастр хужжатини бекор килдириб чана арикчани. Очтиришни иложи борми биз икки кушни сувга кийналаябмиз илтимос ёрдам беринг. Шу муаммо буйича кадастрга хам учраганиз хал булмаган ЮРИСТ ЖАВОБИ: Бу вазиятда аввало, қўшнингиз билан маҳалла фаоллари ўртасида келишишга ҳаракат қилинг, агар бу натижа бермаса, фуқаролик судига сервитут белгилаш бўйича ариза беришингиз мумкин бўлади. Қаранг! Фуқаролик кодексининг 173-моддасида кўчмас мулк (ер участкаси, бошқа кўчмас мулк) эгаси қўшни ер участкасининг эгасидан, зарур ҳолларда эса — бошқа ер участкасининг эгасидан ҳам ўзганинг ер участкасидан чекланган тарзда фойдаланиш (сервитут) ҳуқуқини беришни талаб қилишга ҳақли эканлиги белгиланган. 💁‍♂ Яъни сиздаги вазиятда бўлгани каби қўшнилар бошқа қўшнига тегишли ер участкасидан чекланган миқдорда фойдаланиш ҳуқуқига эга. Бундай фойдаланиш ер участкасидан пиёда ва транспортда ўта олишни таъминлаш, электр узатгич, алоқа ва қувур линияларини ўтказиш ва улардан фойдаланиш, сув билан таъминлаш мақсадида бўлиши мумкин. ✅ Шундай экан, сиздаги вазиятда ҳам қўшнингизнинг еридан ариқ ўтказиш мақсадида фойдаланишингиз мумкин. ☝️Бунда юқорида айтилганидек, мазкур масала аввало тарафларнинг келишуви билан ҳал этилиши мумкин. Сервитутни белгилаш хусусида келиша олинмаса ёки унинг шартларида муросага келинмаса, баҳс сервитут белгилашни талаб қилаётган шахснинг даъвоси бўйича суд томонидан ҳал этилади. Мазкур ҳолатда қўшнингиз ушбу кодекс 173-3-моддасига кўра сиздан ер участкаларидан фойдаланганлик учун мутаносиб ҳақ талаб қилишга ҳақлидир. ✅ Хулоса қиладиган бўлсак, сизлардаги вазиятда низони келишув билан ҳал қилиб бўлмаса, фуқаролик судига қўшнингизнинг ер участкасига нисбатан сервитут белгилаш бўйича ариза беришингиз мумкин. Каналимизга уланинг                 👇👇 👉 ‌‌
Show more ...
Юрист Хизмати Қабул
Ўзингизни қийнаётган барча саҳаларга оид муаммоли саволларингизни юборишингиз мумкин.
37 371
205
#Диққат ❗️«Любой» текширувни қайтариб юбориш учун 1 та лайфхак Тадбиркорларга юбориб қўямиз Каналимизга уланинг                 👇👇 👉 ‌‌

file

36 343
439
📷📷📷📷 🌟 Kimyo fanidan Milliy sertifikat kitobimiz nashrdan chiqdi! 👁 Milliy sertifikat sizga nima beradi? 🤩 A va A+ darajadagi sertifikat abituriyentlar uchun yoʻnalishdan kelib chiqqan holda maksimal ball beradi. 🤩 Shu fandan dars beradigan ustozlarning oyliklariga qoʻshimcha tarzda 20% ustama beradi. ❓ Kitobda nimalar mujassam? 🤩 BMBA talablariga mos ravishda kimyoning umumiy kimyo, anorganik kimyo va organik kimyo boʻlimlaridan tuzilgan ochiq va yopiq test topshiriqlarini oʻz ichiga oladi. 📞 Murojaat uchun: 998932477508 998970204222 ✍️
Show more ...
29 506
104
#Диққат ❗️«Любой» текширувни қайтариб юбориш учун 1 та лайфхак Тадбирклрларга юбориб қўямиз

file

114
0
Ташкилот мурожаатимга жавоб бермаса, қандай чора кўрилади? 4490-савол: U yoki bu tashkilotga murojaat qilinda , lekin tashkilot belgilangan muddat (15 yoki 30 kun ) ichida murojaatga rasman javob bermadi, tashkilot qanday javobgarlikka tortiladi? Бундай ҳолатда кимга мурожаат қилиш керак? ЮРИСТ ЖАВОБИ: ✅ Бундай ҳолатда ўша ташкилотнинг юқори турувчи органига ёки прокуратурага мурожаат қилиш лозим. Қаранг! Жисмоний ва юридик шахсларнинг мурожаатлари тўғрисидаги қонун 28-моддасига кўра ариза ёки шикоят масалани мазмунан ҳал этиши шарт бўлган давлат органига, ташкилотга ёки уларнинг мансабдор шахсига келиб тушган кундан эътиборан ўн беш кун ичида, қўшимча ўрганиш ва (ёки) текшириш, қўшимча ҳужжатларни сўраб олиш талаб этилганда эса, бир ойгача бўлган муддатда кўриб чиқилади. ❗️Шундай экан, мурожаат кўпи билан 30 кун ичида кўриб чиқилиши керак. Қонуннинг 30-моддасида мурожаатлар тўғрисидаги қонунчиликни бузганлик жавобгарликка сабаб бўлиши белгиланган. Бунда Маъмурий жавобгарлик тўғрисидаги кодексининг 43-моддасига асосан жисмоний ва юридик шахсларнинг мурожаатларини кўриб чиқиш муддатларини узрли сабабларсиз бузиш, ёзма ёхуд электрон шаклда жавоб юбормаслик: 👉 мансабдор шахсларга базавий ҳисоблаш миқдорининг бир бараваридан уч бараваригача миқдорда жарима солишга сабаб бўлади. ☝️Мана шу қоидага кўра сиздаги вазиятда ҳам мурожаат қилинган органнинг шу мурожаат билан ишлашга масъул мансабдор шахсига юқоридаги жавобгарлик чораси қўлланилиши мумкин. ✅ Бунинг учун сиз юқорида айтилган органлардан бирига мурожаат қилишингиз лозим. Каналимизга уланинг                 👇👇 👉 ‌‌
Show more ...
Юрист Хизмати Қабул
Ўзингизни қийнаётган барча саҳаларга оид муаммоли саволларингизни юборишингиз мумкин.
34 136
198
Йигит қизга қилган ҳаражатини қайтариб олиши мумкинми? 4489-савол: Assalomu alaykum singlim unashtirilganiga 3 yil bo'ldi. Yaqinda to'y qilamiz deb zaksdan o'tgan edi. Endi to'y malum sabablar bilan to'xtatilyapti. Ikki tomon kelisholmay qolishdi. Yigit qilgan xarajatlarimni to'laysan deb turib olgan. Yani singlim o'qishga vaqti kartasiga pul tashlab turgan. Bundan tashqari qonuniy zaksdan ham o'tib bo'lishgan. Savolim bu vaziyatda pul qaytarib beriladimi va zaks xatini qanday bekor qilish mumkin endi ajrashgan hisoblanadimi? Iltimos tezroq javob bering juda zarur ЮРИСТ ЖАВОБИ: ❗️Бу вазиятда синглингиз пулни қайтармасликка ҳақли. Негаки ҳуқуқий жиҳатдан ўша йигит пулларни синглингизга ҳадя қилган. Фуқаролик кодексининг 502-моддасидаа ҳадя қилувчи ҳадя олувчига ашёни текинга мулк қилиб бериши ёки бериш мажбуриятини олиши ёхуд уни ўзи ёки учинчи шахс олдидаги мулкий мажбуриятдан озод қилиши ёинки озод қилиш мажбуриятини олиши ҳадя сифатида қайд этилган. Сизлардаги ҳолатда ҳам айнан ҳадянинг конструксияси қўлланилган. Яъни йигит маълум миқдордаги маблағни синглингизга ҳадя қилган. Ундан ташқари улар ўртасида қонуний никоҳ қайд этилгандан бошлаб уларнинг биргаликдаги олган даромадлари Оила кодексининг 23-моддасига асосан уларнинг умумий мулки ҳисобланади. Ушбу кодекс 24-моддасига асосан улар умумий мулкдан фойдаланишда, уларни тассаруф этишда (сарфлашда) тенг ҳуқуқлидир. 🙅‍♂ Юқоридаги қоидаларга кўра синглингиздан унга берилган пуллар қайтарилмайди. Уларнинг ҳар иккиси ҳеч қандай низосиз (бизнингча, улар ўртасида низо мавжуд. Пулларни қайтариш ҳақидаги масала ҳам низо) биргаликда ФҲДЁга ариза берган тақдирда Оила кодексининг 42-моддасига асосан ФҲДЁ улар ўртасидаги никоҳни бекор қилади. 💁‍♂ Соддароқ айтганда улар ЗАГС орқали ажрашишлари мумкин. Лекин бизнингча йигит пулни қайтариш ҳақида талаб билан қатъий туриб олса, ФҲДЁ улар ўртасида низо мавжудлиги сабабли никоҳни бекор қилмайди. Бундай ҳолатда никоҳ суд томонидан бекор қилинади. Каналимизга уланинг                 👇👇 👉 ‌‌
Show more ...
Юрист Хизмати Қабул
Ўзингизни қийнаётган барча саҳаларга оид муаммоли саволларингизни юборишингиз мумкин.
32 274
99
Мижозининг сочини унинг хоҳишига зид равишда калга олиб ташлаган сартарош 10 суткага қамалди Жараённи видеога олган фуқарога эса маъмурий жарима жазоси қўлланилган.

file

34 747
100
Мижозининг сочини унинг хоҳишига зид равишда калга олиб ташлаган сартарош 10 суткага қамалди Жараённи видеога олган фуқарога эса маъмурий жарима жазоси қўлланилган.
1
0
Мижозни камситган "крутой" узр сўраб чиқди Асли касби сартарош бўлган шахс вилоятлик болани камситиб, сочини қиртишлаб қўполлик қилгани учун қонунда белгиланган жазога рози экани, мижоз ва унинг яқинларидан узр сўрашини билдирди.

file

file

36 649
143
Машинамга газ баллон ўрнатсам тех паспорт учун 2.7 млн тўлов қиламанми? 4488-савол: Assalom alaykum  men 2023 yil noyabr oyida Damas mashinasini olib davlat ro'yxatidan o'tkazganman  endi 2024 yil aprel oyida gaz baloni o'rnatildi  qayta tex pasport uchun  2 million 700 ming to'lov qilamanmi? ЮРИСТ ЖАВОБИ: Ўзбекистон Республикаси ички ишлар вазирининг 2011 йил 23 ноябрдаги 174-сонли буйруғига иловада тасдиқланган Рўйхатдан ўтказиш давлат рақами белгиларини ва қатъий ҳисобдаги бланкаларни беришда, автомототранспорт воситалари, уларнинг тиркамалари (ярим тиркамалари)ни рўйхатдан ўтказишда, қайта рўйхатдан ўтказишда, мажбурий техник кўрикдан ўтказишда, шунингдек имтиҳонларни олишда ундириладиган тўловлар Ставкалари 7-1-бандига кўра электромобиллар, мототранспорт воситалари ва автотранспорт воситалари тиркамаларидан бошқа барча автотранспорт воситаларини рўйхатдан ўтказиш ва қайта рўйхатдан ўтказиш учун БҲМнинг 6,84 баравари  яъни 2 325 600 сўм тўлаш лозим. ❗️Шунга кўра сиз автотранспорт воситасига газ баллон ўрнатгандан кейин қайта рўйхатдан ўтказсангиз, унда БҲМ 6,84 баравари  яъни 2 325 600 сўм миқдорда тўлов тўлайсиз. Каналимизга уланинг                 👇👇 👉 ‌‌
Show more ...
Юрист Хизмати Қабул
Ўзингизни қийнаётган барча саҳаларга оид муаммоли саволларингизни юборишингиз мумкин.
38 349
276
Истеъмолчи ҳар доим ҳақ! Ижтимоий тармоқларда тарқалган сартарошнинг гаплари мутлақо асоссиз. Мижознинг адашгани сабабли сартарошнинг “Адашсанг айб менда эмас-ку. Нега сен адашасан-у, мен жавоб беришим керак?” қабиладиги гаплари ҳуқуқий мезонларга тўғри келмайди. Ўша йигит сартарашнинг “…кимга айтасан? “Участкавой”га айтасанми? Судга айтасанми? “Мелиса” келадими олдимга?” дегаган гаплари асосида, у айтгандек “Участкавой”га, Судга, “Мелиса”га бориши, шикоят қилиши керак. Аввало, шуни айтиш керакки, мазкур ҳолатда сартарошнинг ўз ҳуқуқбузарлигини тасвирга олиб, уни ижтимоий тармоқларда тарқатгани учун46-2-моддасига асосан (шахсга доир маълумотлар тўғрисидаги қонунчиликни бузиш) базавий ҳисоблаш миқдорининг уч бараваридан беш бараваригача миқдорда жарима солишга сабаб бўлиши белгиланган. Иккинчидан, ушбу кодекс 41-моддасига асосан шахснинг шаъни ва қадр-қимматини қасддан камситгани сабабли базавий ҳисоблаш миқдорининг йигирма бараваридан қирқ бараваригача миқдорда жарима солишга сабаб бўлади. Сартарошга мазкур нормалар билан жавобгарлик чорасини қўллаш учун ўша йигит ИИБга ариза бериши етарли. Ҳуқуқбузарликни тасдиқловчи фактни эса сартарошнинг ўзи тақдим этмоқда. Учинчидан, ўша йигит ўзига етказилган зарарни ва маънавий зиённи ундириши мумкин. г 985-моддасига мувофиқ ғайриқонуний ҳаракат (ҳаракатсизлик) туфайли фуқаронинг шахсига ёки мол-мулкига етказилган зарар, шунингдек юридик шахсга етказилган зарар, шу жумладан бой берилган фойда зарарни етказган шахс томонидан тўлиқ ҳажмда қопланиши лозим. Шунингдек, Фуқаролик кодексининг 1021-моддасига биноан маънавий зарар уни етказувчининг айби бўлган тақдирда, зарар етказувчи томонидан қопланади. Маънавий зарар деганда жабрланувчига қарши содир этилган ҳуқуқбузарлик ҳаракати (ҳаракатсизлик) оқибатида у бошидан кечирган (ўтказган) маънавий ва жисмоний (камситиш, жисмоний оғриқ, зарар кўриш, ноқулайлик ва бошқа) азоблар тушунилади (). Бунда маънавий зарарни қоплаш миқдори жабрланувчига етказилган жисмоний ва маънавий азобларнинг хусусиятига, шунингдек айб товон тўлашга асос бўлган ҳолларда зарар етказувчининг айби даражасига қараб суд томонидан аниқланади (ФК 1022-м). Ҳуқуқ фаоллари, ҳуқуқни муҳофаза қилувчи органлар сартарошга шахсларга қонун борлигини, ишлашини ва фуқароларнинг ҳуқуқлари ҳимоя қилинишини кўрсатиб қўйиши керак.
Show more ...
Сартарош мижознинг сочини унинг хоҳишига зид тарзда калга олиб ташлади
Ижтимоий тармоқларда тарқалган видеода кўринган сартарошнинг бу ҳаракатлари эътирозларга сабаб бўлди. Қонунга кўра унинг қилмиши шахснинг шаъни ва қадр-қимматини қасддан камситиш ҳисобланади ва 13 млн 600 минг сўмгача жаримага сабаб бўлади.
37 864
106
Ҳайдовчилик гувоҳномаси (Право) олиш учун қанча тўлов қилинади? Мазкур муносабат Ўзбекистон Республикаси Ички ишлар вазирининг 2011 йил 23 ноябрдаги 174-сонли буйруғи билан тартибга солинади. Ушбу буйруққа Ички ишлар вазирининг 2023 йил 10 августдаги 396-сонли  билан киритилган ўзгаришга кўра гувоҳнома олиш учун қуйидаги миқдорларда тўлов қилинади. 👉 Назарий имтиҳон топшириш учун - БҲМ 10 фоизи (34 минг сўм); 👉 Ҳар бир қайта назарий имтиҳон топшириш учун - БҲМ 15 фоизи (51 минг сўм); 👉 Автомототранспорт воситаларини, шу жумладан транспорт воситалари таркибини амалий бошқаришда имтиҳон топшириш учун - БҲМ 10 фоизи (34 минг сўм); 👉 Ҳар бир қайта амалий бошқаришда имтиҳон топшириш учун - БҲМ 15 фоизи (51 минг сўм); 👉 Ҳайдовчилик гувоҳномасини бериш учун - БҲМ 70 фоизи (238 минг сўм); 👉 Йўқотилган ҳайдовчилик гувоҳномаси ўрнига вақтинчалик рухсатнома бериш учун - БҲМ 10 фоизи (34 минг сўм); 👉 Чет давлат фуқароси, фуқаролиги бўлмаган шахсларга ва дипломатик хизмат ходимларига ҳайдовчилик гувоҳномаларни бериш учун - БҲМ 3 баравари (1 млн 20 минг сўм) Изоҳ:  2023 йил 1 декабрдан бошлаб эса Ўзбекистон Республикаси Президентининг 17.11.2023 йилдаги ПФ-196-сон  Фармонига кўра базавий ҳисоблаш миқдори 340 000 сўм этиб белгиланган. Ўзбекистон Республикаси Президентининг 2020 йил 22-октабрдаги ПҚ-4871-сон “Фавқулодда вазиятлардан жабр кўрган фуқароларни ижтимоий ҳимоя қилишнинг қўшимча чоралари тўғрисида”ги қарорига асосан, Ўзбекистон Республикаси ва хорижий давлатлар ҳудудида табиий ва техноген хусусиятли фавқулодда вазиятлар, шу жумладан маиший тусдаги офатлар (турар, нотурар-жойнинг ёнғини ёки қулаши, транспорт воситасининг ёнғини) юз берганлиги оқибатида фуқаронинг айбисиз йўқотилган ёки яроқсиз ҳолга келган ҳайдовчилик гувоҳномасининг дубликатини бериш учун тўлов ундирилмайди. Батафсил: лойиҳамизда
Show more ...
Onlayn yuridik xizmatlar Yuridik Maslahat Malakali yurist maslahati Xizmatdan foydalanish Shartnoma Tayyorlash Barcha turdagi shartnomani yaratamiz Xizmatdan foydalanish Aliment undirish Bola uchun Aliment undiramiz Xizmatdan foydalanish Mediatorlik Nizolarni sudsiz hal qilib beramiz Xizmatdan foydalanish Da’vo arizalari Fuqarolik va iqtisodiy masalalarda da’vo arizalari yozamiz Xizmatdan foydalanish Boshqa xizmatlar Boshqa masalada yordam kerakmi unda biz bilan bog’laning […]
44 691
601
Айбдор вафот этса, зарарни ундириш мумкинми? 4487-савол: Avtohalokat natijasida avtomashinamiz tamirga muhtoj aybdor avtomashina egasi voqea joyida vafot topdi aybsiz avtomoshinani kim tamirlab beradi qonuniy kimga murojot qilamiz. Jami harajat 34 mln aybdor mashina sugurtasi 14 mln dan ortiq chiqarolmedikan qolgan 20 mln ni kimdan olaman ЮРИСТ ЖАВОБИ: ✅ Бу вазиятда зарарни қоплаш бўйича сиз ўша шахснинг меросхўрларига даъво қилишингиз мумкин. 💁‍♂ Буни қуйидагича тушунтирамиз. Фуқаролик кодексининг 351-моддасига кўра агар мажбуриятни қарздорнинг шахсий иштирокисиз бажариш мумкин бўлмаса ёки мажбурият бошқача тарзда қарздорнинг шахси билан чамбарчас боғлиқ бўлса, қарздор вафот этиши билан мажбурият бекор бўлади. ☝️Эътибор берган бўлсангиз, қоидада қарздорнинг шахси билан боғлиқ мажбуриятлар бекор бўлиши айтилмоқда. Бундай мажбуриятлар қаторига алимент тўлаш, ҳаёт ёки соғлиққа етказилган зарарни тўлаш, бу мажбуриятни фақат шу қарздор бажара оладиган бошқа ҳаракатлар киради. Сиздаги вазиятда мажбурият мулкий мажбурият тоифасига киради. Қаранг! Фуқаролик кодексининг 1113-моддасига биноан мерос очилган пайтда мерос қолдирувчига тегишли бўлган, унинг ўлимидан кейин ҳам бекор бўлмайдиган барча ҳуқуқ ва мажбуриятлар мерос таркибига киради. ☝️Мана шу қоидага кўра сиздаги вазиятда сизнинг мулкингизга зарар етказган шахснинг мажбуриятлари универсал ҳуқуқий ворислик тарзида ҳуқуқлари билан биргаликда унинг ворисларига ўтади. Бунда ушбу кодекс 1156-моддасига асосан мерос қолдирувчининг кредиторлари васиятномани ижро этувчига (меросни бошқарувчига) ёки меросхўрларга мерос қолдирувчининг мажбуриятларидан келиб чиқадиган ўз талабларини қўйишга ҳақлидирлар. Бу ҳолда меросхўрларнинг ҳар бири ўзига теккан мол-мулкнинг қиймати доирасида солидар қарздорлар сифатида жавобгар бўладилар. ✅ Шунга кўра сиз ҳам фуқаролик судига даъво қилиб, зарар етказган шахснинг ворисларидан зарарни ундиришни сўрашингиз мумкин. Каналимизга уланинг                 👇👇 👉 ‌‌
Show more ...
Юрист Хизмати Қабул
Ўзингизни қийнаётган барча саҳаларга оид муаммоли саволларингизни юборишингиз мумкин.
38 197
62
#Биласизми ❗️Ўқувчи дарсда дафтар тутмагани учун ота-она жавобгарликка тортилади Ўқувчи дафтар тутмагани учун ўқитувчини эмас, ота-онани жавобгарликка тортиш керак. 1⃣ Биринчидан, Таълим тўғрисидаги қонун 51-моддасида вояга етмаган таълим олувчиларнинг ота-онаси ва бошқа қонуний вакилларининг бир қатор мажбуриятлари қайд этилган бўлиб, унга кўра юқоридаги шахслар: 👉 ўз болаларини инсонпарварлик, ватанпарварлик, меҳнатсеварлик, маънавий, миллий ва умуминсоний қадриятларни ҳурмат қилиш руҳида тарбиялаши; 👉 ўз болаларининг интеллектуал, маънавий ва жисмоний ривожланиши учун шарт-шароитлар яратиши; 👉 таълим-тарбия жараёни иштирокчиларининг шаъни ва қадр-қимматини ҳурмат қилиши шарт. Айнан мана шу мажбуриятлар ўқувчининг қонуний вакилларининг ўқувчининг ҳуқуққа ҳилоф ҳаракатлари учун жавобгарликни ўз зиммасига олишини англатади. 2⃣ Иккинчидан, Халқ таълими вазирлигининг 01.07.2021 йил 206-сонли буйруғига асосан тасдиқланган ўқитувчиларнинг лавозим мажбуриятларида ўқитувчининг ўқувчиларга дафтар туттириш мажбурияти қайд этилмаган. 💁‍♂ Бундай ҳолатда ота-онага Маъмурий жавобгарлик тўғрисидаги кодекс 47-модддаси иккинчи қисми билан жавобгарлик чораси қўлланилади. Яъни Худди шундай ҳуқуқбузарликлар маъмурий жазо чораси қўлланилганидан кейин бир йил давомида такрор содир этилган бўлса: ➖ базавий ҳисоблаш миқдорининг 5 бараваридан 10 бараваригача миқдорда жарима солишга сабаб бўлади. Каналимизга уланинг                 👇👇 👉 ‌‌
Show more ...
Юрист Хизмати Қабул
Ўзингизни қийнаётган барча саҳаларга оид муаммоли саволларингизни юборишингиз мумкин.
120 376
2 081
Биз ҳақимизда қисқача...
31 334
3
Жиянимни отиб қўйишди, зарарни ундирса бўладими? 4486-савол: Assalomu alaykum. Jiyanim Toshkent davlat texnika universitetida kontrakt asosida  3-kurs talabasi (harbiylashtirilgan kafedrada ham oqiydi). Dalada ovchilar travmatik miltiq bilan otib qoygan jiyanimni. Shundan keyin jiyanim  6-soatlik operatsiya qilindi. Javobgar shaxsni ichki ishlar xodimlari olib ketgan. Jiyanim to'lov kantraktini va boshqa xarajatlarini darsdan keyin ishlab topardi. Hozirda Nomalum muddatga ishga yaroqsiz bo'lib qoldi. Ushbu hodisada jiyanimga javobgar shaxs tomonidan qanday moliyaviy yordam korsatilishi kerak(albatta qonuniy tomonlama). ЮРИСТ ЖАВОБИ: Фуқаролик кодексининг 234-моддасида мажбуриятларнинг шартномалардан ёки шартномалардан ташқари мажбуриятлардан келиб чиқиши белгиланган. Сиздаги вазиятда ҳам мажбурият (етказилган зарарни қолаш) шартномадан ташқари асосда (бошқа шахсга зарар етказиш натижасида) келиб чиқмода. Ушбу кодекс 985-моддасига мувофиқ ғайриқонуний ҳаракат (ҳаракатсизлик) туфайли фуқаронинг шахсига ёки мол-мулкига етказилган зарар, шунингдек юридик шахсга етказилган зарар, шу жумладан бой берилган фойда зарарни етказган шахс томонидан тўлиқ ҳажмда қопланиши лозим. ☝️Айнан мана шу қоида жиянингизга ўзига етказилган зарарни қоплашни талаб қилиш ҳуқуқини беради. ❓Энди асосий масалага ўтсак, зарар қандай аниқланади. Бунда Фуқаролик кодексининг 14-моддаси қоидалари қўлланилади. Ушбу норманинг биринчи қисмида ҳуқуқи бузилган шахс ўзига етказилган зарарнинг тўла қопланишини талаб қилиши мумкинлиги белгиланган. ✅ Шунга кўра сиздаги вазиятда ҳам жиянингиз ўзига етказилган зарарни тўлиқ қоплашни талаб қилишга ҳақли. Бунда ушбу норма иккинчи қисмига биноан зарар деганда, ҳуқуқи бузилган шахснинг бузилган ҳуқуқини тиклаш учун қилган ёки қилиши лозим бўлган харажатлари, унинг мол-мулки йўқолиши ёки шикастланиши (ҳақиқий зарар), шунингдек бу шахс ўз ҳуқуқлари бузилмаганида одатдаги фуқаролик муомаласи шароитида олиши мумкин бўлган, лекин ололмай қолган даромадлари (бой берилган фойда) тушунилади. ☝️Мана шу қоидага биноан зарар миқдори жиянингиз соғлигига етказилган зарар, унинг оладиган қўшимча даромадларининг ҳақиқий қиймати саналади. Бу ҳолатда жиянингиз фуқаролик судига мурожаат қилиб, зарарни қоплашни сўрашингиз керак. Каналимизга уланинг                 👇👇 👉 ‌‌
Show more ...
Юрист Хизмати Қабул
Ўзингизни қийнаётган барча саҳаларга оид муаммоли саволларингизни юборишингиз мумкин.
34 320
29
❗️"ГАИ"дан келадиган жаримани тўламаса ҳам бўлади, қачонки... 4485-савол: Ассалому алайкум, менга кеча ДЙХХХ (Гаи) дан радарга тушганлигим хакида жарима когозлари келди, лекин унда мен 2022 йил 30 сентябр куни, 2023 йил сентябрда ва яна бошка ойларда, туманимиздаги асосий магистрал йўлда автомашинамда тезликни оширганим учун тайинланган экан, лекин буларни барчасини 30.03.2024 йил куни карорлаштириб, энди менга почта оркали кеча юборишди, қисқача қилиб айтганда, 2022 ва 2023 йиллар жармасини энди карорлаштириб юборишди. Саволим шуки, коидани бузган кундан, то қарор чикган кунгача муддат канча булиши керак, нега энди менга жамланган ҳолда энди юборишяпти, бундан ташкари бази жарима қоғозларида QR код мавжуд эмас (сканер киладиган туртбурчакли), кейин мен бир кунни ўзида ўша йулда ўша радарда кайта тушганман, бир кунни санаси билан булган жарималарни биттасини тулаш керак эмасми, илтимос шуларга иложи борича конуний жавоб берсангиз ЮРИСТ ЖАВОБИ: Ички ишлар вазирининг 2011 йил 31 майдаги 68-сон буйруғи билан тасдиқланган ( рўйхат рақами № 2240) «Йўл ҳаракати қоидаларининг бузилишига доир маъмурий ишларни кўриб чиқиш тартиби тўғрисида»ги йўриқноманинг 69-бандига мувофиқ агар маъмурий жазо қўлланиш тўғрисидаги қарор чиқарилган кундан бошлаб уч ой мобайнида ижрога қаратилмаган бўлса (қарор нусхаси ҳуқуқбузарга топширилмаган, ҳуқуқбузарнинг ишлаш, яшаш жойига ёки давлат ижрочисига юборилмаган бўлса ва ҳ. к.), у ҳолда бундай қарор ижро этилмайди. ❗️Бундан кўриниб турибдики, жарима солиш ҳақидаги қарор нусхаси МЖТК 309-1-моддасида белгиланган муддатда ҳуқуқбузарга етказилмаса, унда бу қарор ижрога қаратилмайди. 💁‍♂ Соддароқ айтганда, агар қарор нусхаси юборилмаган бўлса, унда жаримани тўламаса ҳам бўлади. ✅ Шунга кўра сиз ушбу жарималарни суд орқали бекор қилишингиз мумкин. ❗️Бунинг учун маъмурий судга жарима тайинлаш ҳақидаги қарорни бекор қилиш ҳақида ариза беришингиз керак. Каналимизга уланинг                 👇👇 👉 ‌‌
Show more ...
42 205
371
❗️1 майдан "свет"-газ учун янги тарифлар жорий қилинади ва ижтимоий норма жорий этилади Ҳукумат 2 йилдан буён "ана бўлади, мана бўлади", дея кутилаётган қарорни ниҳоят қабул қилибди. Энергетика соҳасидаги нарх сиёсатини ислоҳ қилиш ҳақида аввал ҳам кўп гапирилган, лекин турли сабабларга кўра, орқага сурилиб келинаётганди. Энди қарорга кўра, жорий йилнинг 1 майидан ижтимоий норма жорий этилади. Тўғри, қарорда айнан "ижтимоий норма" деган жумла ишлатилмаган, ўрнига “базавий меъёр механизми" дейилган. Лекин мазмуни бир хил. ❓Ижтимоий норма нима дегани? Ижтимоий норма - бу аҳоли томонидан имтиёзли тариф бўйича тўланадиган маълум миқдордаги энергия тушунилади. Яъни, муайян белгиланган миқдоргача нисбатан пастроқ (имтиёзли) нархда сотилади, етмагани давлат бюджетидан субсидия қилинади. Бунда тарифлар бир нечта категорияга бўлинган ва қанча ишлатилганига қараб, нархлар ҳам ўзгаради. Ким кўп ишлатса, шунча кўп тўлайди, кам ишлатганга имтиёзли нарх берилади. 💡Електр энергиясида категориялар ва нархлар қуйидагича: ойига 200 кВт·соатгача – 450 сўм; ойига 201 кВт·соатдан 1 000 кВт·соатгача – 900 сўм; ойига 1 001 кВт·соатдан 5 000 кВт·соатгача – 1 350 сўм; ойига 5 001 кВт·соатдан 10 000 кВт·соатгача – 1 575 сўм; ойига 10 000 кВт·соат ва ундан юқори – 1 800 сўмдан. 🔥Газда категориялар ва нархлар қуйидагича: Коммунал-маиший эҳтиёжлар учун аҳолига мавсумга қараб нарх белгиланмоқда. Хусусан, қишки мавсумда (ноябрь-февраль ойлари) давомида бир ойдаги истеъмолдан келиб чиқиб тегишли нарх белгиланмоқда: ойига 500 куб метргача – 650 сўм; ойига 501 куб метрдан 2 500 куб метргача – 1 500 сўм; ойига 2 501 куб метрдан 5 000 куб метргача – 1 950 сўм; ойига 5 001 куб метрдан 10 000 куб метргача – 2 275 сўм; ойига 10 001 куб метр ва ундан юқори – 2 600 сўмдан. Март – октябрь ойларида истеъмолчилар бир ойдаги истеъмолидан келиб чиққан ҳолда қуйидагича ҳақ тўлайди: ойига 100 куб метргача – 650 сўм; ойига 101 куб метрдан 2 500 куб метргача – 1 500 сўм; ойига 2 501 куб метрдан 5 000 куб метргача – 1 950 сўм; ойига 5 001 куб метрдан 10 000 куб метргача – 2 275 сўм; ойига 10 001 куб метр ва ундан юқори – 2 600 сўмдан. ❓Еҳтиёжманд аҳолига қандай ёрдам берилади? Эҳтиёжманд аҳолини қўллаб–қувватлаш мақсадида давлат бюджетидан 1 трлн сўм маблағ ажратилади. Ҳукумат қарорига кўра, эҳтиёжманд оилаларга қуйидаги 2 та ёрдам кўрсатиш назарда тутилган: 1) иситиш мавсумида (ноябрь – февраль ойларида) эҳтиёжманд оилаларга 270 минг сўм миқдорида моддий ёрдам берилади; 2) ижтимоий нормадан қуйидаги миқдоргача ошиқча ишлатилган "свет" ва газ тўлови давлат бюджетидан компенсация қилинади: ➖ табиий газ учун – ойига базавий меъёрдан ортиқ фойдаланилган 250 куб метргача;електр энергияси учун – ойига базавий меъёрдан ортиқ фойдаланилган 150 кВт·соатгача. 👉 👈
Show more ...
86 082
2 077
❗️Қисқартиришга тушган ходимлар имтиёзли пенсияга чиқиш ҳуқуқига эга Пенсияга чиқишда иш стажидан ташқари ёшга оид меёр ҳам белгиланади. Умумий қоида бўйича  ёшга доир пенсия олиш ҳуқуқига: 👉 эркаклар — 60 ёшга тўлганда ва иш стажлари камида 25 йил бўлган тақдирда; 👉 аёллар — 55 ёшга тўлганда ва иш стажлари камида 20 йил бўлган тақдирда эга бўладилар. Бу Фуқароларнинг давлат пенсия таъминоти тўғрисидаги қонуни 7-моддасида мустаҳкамланган. Шуни айтиш керакки, Қонуннинг 14-моддасига кўра қисқартириш ҳолатлардаги эркаклар 58 ёшдан пенсияга чиқиши мумкинлиги ҳақидаги имтиёз белгиланган. Унга кўра технологиядаги, ишлаб чиқариш ва меҳнатни ташкил этишдаги ўзгаришлар, ходимлар сони (штати) ёки иш хусусиятининг ўзгаришига олиб келган ишлар ҳажмининг қисқарганлиги ёхуд корхонанинг тугатилганлиги муносабати билан ишдан озод этилган ва ишсиз деб эътироф этилган шахслар: эркаклар — 58 ёшга тўлганда ва иш стажи камида 25 йил бўлган тақдирда; аёллар — 53 ёшга тўлганда ва иш стажи камида 20 йил бўлган тақдирда пенсия олиш ҳуқуқига эга бўладилар. ✅ Шунга кўра корхонага ходимлар сони қисқарганда ёки корхона бутунлай тугатилганда унда ишлаётган юқоридаги тоифадаги ходимлар имтиёзли равишда пенсияга чиқишга ҳақлидир.
Show more ...
46 809
453
⚡️Инглиз тили ўқитувчиларига 50 фоиз устама берадиган сертификатлар РЎЙХАТИ (2024) Ўзбекистон Республикаси Мактабгача ва мактаб таълими вазирлиги ҳамда Иқтисодиёт ва молия вазирлигининг 2023 йил “28”-июлдаги , қуйидаги сертификатларнинг тегишли даражалари учун ИНГЛИЗ ТИЛИ ЎҚИТУВЧИЛАРИнинг базавий тариф ставкаларига нисбатан *50 фоизлик устамалар* тўлаш . Улар: 1. Миллий сертификат (CEFR, Multi level) - С1 2. IELTS - 7.0 3. TOEFL iBT - 95 4. TOEFL ITP - 627 (Speaking - 64) 5. Cambridge Assessment English (CAE) - 180 6. TESOL, TESL, TEFL(*вазирлик талаби) 7. CELTA (**вазирлик талаби) 8. DELTA (**вазирлик талаби) Module Pass, Pass A, Pass B 9. TKT Module 1-2-3 - Band 3-4 10. Linguaskill (01.08.2023 га қадар олинган) - C1 11. Aptis (01.08.2023 га қадар олинган) - C 12. iTEP (01.08.2023 га қадар олинган) - C1 13. TOEIC (01.08.2023 га қадар олинган) - 1305 * - Вазирлик томондан қарор 5-банди а ёки б талабларини бажарганда ** - Вазирлик томондан қарор 6-банди а ёки б талабларини бажарганда ☝️ Таълим соҳасида хизмат кўрсатувчи ваколатли халқаро ташкилотларнинг расмий рўйхатидан ўтган тест марказларига (authorized test centers) ташриф буюрмасдан, ташкилот вакилларининг бевосита назорати остида топширмасдан, масофадан (Home Edition, Remote testing ва бошқа) онлайн олинган сертификатлар учун УСТАМА ТЎЛАНМАЙДИ!
Show more ...
54 762
1 109
⚡️Инглиз тили ўқитувчиларига 50 фоиз устама берадиган сертификатлар РЎЙХАТИ (2024) Ўзбекистон Республикаси Мактабгача ва мактаб таълими вазирлиги ҳамда Иқтисодиёт ва молия вазирлигининг 2023 йил “28”-июлдаги , қуйидаги сертификатларнинг тегишли даражалари учун ИНГЛИЗ ТИЛИ ЎҚИТУВЧИЛАРИнинг базавий тариф ставкаларига нисбатан *50 фоизлик устамалар* тўлаш . Улар: 1. Миллий сертификат (CEFR, Multi level) - С1 2. IELTS - 7.0 3. TOEFL iBT - 95 4. TOEFL ITP - 627 (Speaking - 64) 5. Cambridge Assessment English (CAE) - 180 6. TESOL, TESL, TEFL(*вазирлик талаби) 7. CELTA (**вазирлик талаби) 8. DELTA (**вазирлик талаби) Module Pass, Pass A, Pass B 9. TKT Module 1-2-3 - Band 3-4 10. Linguaskill (01.08.2023 га қадар олинган) - C1 11. Aptis (01.08.2023 га қадар олинган) - C 12. iTEP (01.08.2023 га қадар олинган) - C1 13. TOEIC (01.08.2023 га қадар олинган) - 1305 * - Вазирлик томондан қарор 5-банди а ёки б талабларини бажарганда ** - Вазирлик томондан қарор 6-банди а ёки б талабларини бажарганда ☝️ Таълим соҳасида хизмат кўрсатувчи ваколатли халқаро ташкилотларнинг расмий рўйхатидан ўтган тест марказларига (authorized test centers) ташриф буюрмасдан, ташкилот вакилларининг бевосита назорати остида топширмасдан, масофадан (Home Edition, Remote testing ва бошқа) онлайн олинган сертификатлар учун УСТАМА ТЎЛАНМАЙДИ!
Show more ...
723
23
Ер тоифасини қандай ўзгартирса бўлади? 4484-савол: Менга ажратилган ер участкаси махалла (шахар ичида курилган уйлар ва янги участкалар) худудидан ажратилган (кишлок хужалиги ёки сугориладигаан ерлар да эмас). Архитектура-режалаштириш топшириги  хужжати буйича: Объект жойлашган худуд кайси зонага мансублиги -"ахоли пункти" деб курсатилган. Агарда шу зонага мансуб булса, унда ер тоифасини узгартириш мумкинми? Илтимос маслахат берсангиз. Мен шу жойдан билишга харакат килдим https://advice.uz/uz/document/2750\ ЮРИСТ ЖАВОБИ: Сиз кўрган маълумотлар бино тоифасини ўзгартириш ҳақида. Лекин улар ер тоифасига нисбатан татбиқ этилмайди. Қаранг! Ер кодексининг 9-моддасига кўра ерлар асосий фойдаланиш мақсадига қараб ер фонди тоифаларига бўлинади. Ушбу кодекс 8-моддасига кўра шаҳарлар ва посёлкалар, шунингдек қишлоқ аҳоли пунктлари чегараси доирасидаги ерлар аҳоли пунктларининг (шаҳарлар, посёлкалар ва қишлоқ аҳоли пунктларининг) ерлари тоифасига киради. Энди қаранг! мазкур тоифани ўзгартириш тартиби 9-моддада ёзилган. Ушбу модданинг тўртинчи қисмига кўра ерларни ер фонди тоифаларига бўлиш ва бир тоифадан бошқасига ўтказиш вилоятлар ҳамда Тошкент шаҳар ҳокимлари томонидан амалга оширилади. 💁‍♂ Бунда ерни ер фондининг бир тоифасидан бошқасига ўтказиш ерлардан асосий фойдаланиш мақсади ўзгарган тақдирда амалга оширилади. ✅ Шунга кўра сизлар ҳам ерни тоифасини ўзгартириш масаласида вилоят ҳокимига мурожаат қилишингиз лозим. Каналимизга уланинг                 👇👇 👉 ‌‌
Show more ...
Юрист Хизмати Қабул
Ўзингизни қийнаётган барча саҳаларга оид муаммоли саволларингизни юборишингиз мумкин.
36 987
46
⚡️Юрист хизмати ўз лойиҳасини янада кенгайтирди – Онлайн Юридик хизматлар портали. Ушбу портал орқали қуйидаги хизматларни таклиф этамиз: ➖ ҳуқуқий соҳага оид ҳар қандай масалага онлайн ва офлайн консултация олиш; ➖ адвокатлик ва медиаторлик хизматидан фойдаланиш; ➖ юридик шахс ташкил этиш, қайта ташкил этиш ва тугатиш; ➖ юридик шахсларнинг таъсис ҳужжатларини ишлаб чиқиш; ➖ шартнома лойиҳаларини тайёрлаш ва шартномаларни ҳуқуқий экспертизадан ўтказиш. ❕Алоҳида таклиф: Фирмалар билан шартнома асосида ишлашимизни маълум қиламиз. Бизнинг жамоа малакали адвокатлар ва юристлар, профессионал медиаторлар ҳамда юридик фанлар доктори, профессорлардан ташкил топган.  Шунингдек, Юридик Блогда кўплаб қизиқарли мақолалар, қонунчилик янгиликлари ва ўзингизни қийнаётган ҳуқуқий масалаларга жавоб топишингиз мумкин.  👉 @Yurist_Xizmati
Show more ...
34 311
58
ШТАТЛАР ТЎҒРИ АЖРАТИЛГАНМИ? 4483-савол: Ҳозирги кунда мактаб-интернатимизда 170 бола таълим-тарбия олмоқда. Молия вазирлигига  юқоридаги норма асосида 1.5 ставка шифокор-ортопед врач, 3.25 ставка жисмоний тарбия билан даволаш бўйича йўриқчи, 3.25 ставка массажчи, 1.75 ставка гипсчи, 5.25 ставка тиббий ҳамшира ва 3.25 ставка санитарка шатларига смета ишлаб молия вазирлига тақдим этилди. Лекин улар томонидан 100 болага штат ажратилиб, қолган 70 болага штат ажратилмади. Нимага деганда меъёрда ҳар 100 болага деб кўратилган лигини, 70 бола 100 дан камлигини сабаб қилишмоқда. Шу ҳолат тўғри талқин қилинганми. Жавоб учун рахмат. ЮРИСТ ЖАВОБИ:Штатлар сони тўғри ажратилган. Негаки 406-сон буйруқда ҳар 100 нафар болага меёр белгиланган. Молия органи ушбу меёрни ҳисобга олган ҳолда 100 нафар бола ҳисобига:1 штат бирлиги – Шифокор - ортопед ➖ 2 штат бирлиги – Жисмоний тарбия билан даволаш бўйича йўриқчи ➖ 2 штат бирлиги – Массажчи  ➖ 1 штат бирлиги – Гипс бўйича техник ➖ 3 штат бирлиги – Тиббий ҳамшира  ➖ 2 штат бирлиги – Санитарка штат бирлиги ажратган бўлса, 70 нафар бола ҳисобига қўшимча: 0.5 штат бирлиги – Шифокор - ортопед ➖ 3.25 штат бирлиги – Жисмоний тарбия билан даволаш бўйича йўриқчи ➖ 3.25 штат бирлиги – Массажчи  ➖ 1,75 штат бирлиги – Гипс бўйича техник ➖ 5.25 штат бирлиги – Тиббий ҳамшира  ➖ 3.25 штат бирлиги – Санитарка штат бирлиги ажратган. Бу эса қонуний ҳисобланади. Каналимизга уланинг                 👇👇 👉 ‌‌
Show more ...
Юрист Хизмати Қабул
Ўзингизни қийнаётган барча саҳаларга оид муаммоли саволларингизни юборишингиз мумкин.
38 461
68
Ўқитувчи дарсни масъулятсизлик билан ўтса, унга қандай чора кўриш мумкин? 4482-савол: Ассалому алайкум ота оналар укитувчининг укувчига килаётган нохаклиги куйилган нохак бахо масулиятсиз утилаётган дарслари дарс вактида телефон титкилаб утириши ва купол муомиласи устидан каерга арз килади? ЮРИСТ ЖАВОБИ: Мактабгача ва мактаб таълими вазирининг 2023-йил 20 - декабрдаги 565-сонли буйруғига 37-иловада тасдиқланган Умумий ўрта таълим муассасалари (айрим фанлар чуқур ўрганиладиган ихтисослаштирилган муассасалар) ходимларининг лавозим мажбуриятларига асосан ўқитувчи: 👉 дарс машғулотларини умумий ўрта таълим муассасаси раҳбарияти томонидан тасдиқланган дарс жадвали ҳамда тақвим-мавзу режа асосида олиб бориш; 👉 ўз фани бўйича белгиланган методик кунда ўзининг касбий маҳоратини ошириш мақсадида ўз устида мустақил ишлаш, илғор таълим методларини ўрганиш ва доимий равишда малакасини ошириб бориш чораларини кўриш; 👉 ўқувчилар ҳуқуқларига ва болаларни муҳофаза қилишга оид қонунчилик талабларига риоя қилиш; 👉 таълим-тарбия жараёни иштирокчиларининг шаъни, қадр-қиммати ва ишчанлик обрўсини ҳурмат қилиш каби мажбуриятларни бажариши керак. ❗️Агар ўқитувчи бу мажбуриятларни бажармаётган бўлса, унда директорга мурожаат қилиш лозим. Директор мазкур масалани кўриб чиқиб, унга нисбатан интизомий таъсир чорасини қўллашга ҳақли. Бунда Меҳнат кодексининг 312-моддасига мувофиқ меҳнат интизомини бузганлиги учун иш берувчи ходимга қуйидаги интизомий жазо чораларини қўллашга ҳақли: 1) ҳайфсан; 2) ўртача ойлик иш ҳақининг ўттиз фоизидан кўп бўлмаган миқдорда жарима. Ички меҳнат тартиби қоидаларида ходимга ўртача ойлик иш ҳақининг эллик фоизидан кўп бўлмаган миқдорда жарима солиниши ҳоллари назарда тутилиши мумкин. 3) меҳнат шартномасини бекор қилиш. ☝️Ушбу қоиданинг мазмунига кўра ходим меҳнат интизомини бузганда унга юқоридаги таъсир чораларидан бири қўлланилади.
Show more ...
57 587
685
Ўқувчини қуйи синфдан юқори синфга кўчириш мумкинми? 4481-савол: Ассалому алайкум! Эътиборсиз колдирмай жавоб берганингиз учун миннатдорчилик билдирмокчиман. Углом 2011 йилда тугилган, хозир 6 синфда укийди, синфдошларининг аксарияти 2012 йил, унинг тенгдошлар 7синфда укишади. Укишлари яхши, табелида  бирорта хам 4 бахоси йук, синфдан синфга кучиришни иложи борми? ЮРИСТ ЖАВОБИ: ❌ Амалдаги қонунчиликка мувофиқ кўчира олмайсиз деб ҳисоблаймиз. Вазирлар Маҳкамасининг 2017 йил 15 мартдаги 140-сон қарорига 1-иловада тасдиқланган Умумий ўрта таълим тўғрисидаги Низом қоидаларида ўқувчиларни мактабга қабул қилиш, ўқишини кўчириш, бир синфдан бошқа синфга ўтказиш (яъни 6-А синфдан 6-Б синфга ўтказиш), бошқа мактабга ўқишни кўчириш тартиблари белгиланган. ❗️Бироқ ўқувчини қуйи синфдан юқори синфга кўчириш қоидалари белгиланмаган. Шунинг учун ўқишни кўчира олмайсиз. Каналимизга уланинг                 👇👇 👉 ‌‌
Show more ...
Юрист Хизмати Қабул
Ўзингизни қийнаётган барча саҳаларга оид муаммоли саволларингизни юборишингиз мумкин.
41 381
88
Отамдан қолган уйни ҳужжати йўқ, уни қандай номимизга ўтказсак бўлади? 4480-савол: Assalomu alaykum. Meni dadam olamdan o‘tgan dadamning nomida uy mashena bor natariusga borgan edik meros rasmiyashtirgani  natarius uyning egalik huquqi yo‘qligi uchun meros huquqi ochmadi.  Biz yashab turgan uy qurilganinga 30yildan oshdi kadastr qilingan egalik huquqi yo‘q. Hozirda egalik huquqini hech qaysi tashkilot bermayabdi uy bilan mashenani dakument qilishimiz uchun nima qilishimiz kerak. Qonuniy yo‘li bo‘lsa tushuntirsangiz biz kimga uchraymiz qanday qilib dakument qilsak bo‘ladi uy bilan mashenani. Javobingiz uchun rahmat. ЮРИСТ ЖАВОБИ: Кўчмас мулкка бўлган ҳуқуқларни давлат рўйхатидан ўтказиш тўғрисидаги қонун 31-моддасига асосан 1998-йилнинг 1-июлидан 2021-йилнинг 1-августига қадар қабул қилинган ер участкасига бўлган ҳуқуқларни белгиловчи ҳужжатлар қуйидагилардан иборат: 👉 ер участкасига мерос қилиб қолдириладиган умрбод эгалик қилиш ҳуқуқини давлат рўйхатидан ўтказиш учун — маҳаллий давлат ҳокимияти органларининг ер участкасини бериш ҳақида ўз ваколатлари доирасида қабул қилинган қарорлари, нотариал тасдиқланган мерос қилиб олиш ҳуқуқи тўғрисидаги гувоҳнома, ушбу ҳуқуқнинг ауксионда сотиб олинганлигига асосан туман (шаҳар) ҳокими томонидан берилган давлат ордери асос бўлади. ❗️Шу қоидага кўра сизлардаги вазиятда ер участкасига нисбатан ушбу ҳуқуқларни тасдиқловчи ҳужжатлар мавжуд бўлмаса, нотариус ер участкасига нисбатан меросга бўлган ҳуқуқ тўғрисидаги гувоҳнома бера олмайди. Бу вазиятда сизлар фуқаролик судига эгалик ҳуқуқини белгилаш бўйича даъво аризаси билан мурожаат қилишингиз лозим. Қаранг! Фуқаролик кодекси 187-моддасига мувофиқ мулкдор бўлмаган, лекин кўчмас мол-мулкка ўн беш йил давомида ёки бошқа мол-мулкка беш йил давомида ўзиники каби ҳалол, ошкора ва узлуксиз эгалик қилган шахс бу мол-мулкка мулк ҳуқуқини олади (эгалик қилиш ҳуқуқини вужудга келтирувчи муддат). Қоиданинг мазмунига мувофиқ фуқаро ўзига тегишли бўлмаган мулкдан 15 йил давомида фойдаланса, уни эскиришга эмас, балки яхшиланишига эришса, бу мулкдан фойдаланиши ҳақида бошқалар (қўни-қўшни, МФЙ ва ҳ.к.) хабардор бўлса ва энг асосийси фойдаланиш муддати узулишсиз бўлса, (яъни 10 йил мулкдан фойдаланиб орада 1 ёки 2 йил фойдаланмай кейин яна фойдаланган бўлмаслиги) шахсда шу мулкка нисбатан мулк ҳуқуқини олиш мумкин. Мазкур кодекс 1995-йилда қабул қилинган ва шу даврдан бошлаб юзага келадиган ижтимоий муносабатларни тартибга солиб келади. Шундай экан сиздаги вазиятда 1990-йилдан бери уйда (ҳақиқатда мулкдори бўлмаган) яшаб, ундан фойдаланиб келаётган фуқарода уйга мулк ҳуқуқини олиш мумкинлигини кўрсатади. Энди қаранг! кўчмас ва бошқа мол-мулкка давлат рўйхатидан ўтказилиши шарт бўлган мулк ҳуқуқи ушбу мол-мулкка эгалик қилиш ҳуқуқини вужудга келтирувчи муддат туфайли олган шахсда ана шундай рўйхатдан ўтказилган пайтдан бошлаб вужудга келади. Бундан кўриниб турибдики, уйга нисбатан мулк ҳуқуқи давлат рўйхатидан ўтказилган пайтдан бошлаб вужудга келади. ☝️Ушбу модданинг тўртинчи қисмида эгалик қилиш муддатини важ қилиб келтирадиган шахс ўзи ҳуқуқий вориси бўлган шахснинг бу мол-мулкка эгалик қилиб келган барча вақтини ўзининг эгалик қилиш вақтига қўшиши мумкинлиги айтилмоқда. 💁‍♂ Қоидани қуйидаги мисол билан тушунтирамиз. Айтайлик бир фуқаро ўзиники бўлмаган уйда 2003 йилдан яшашни бошлади. Солиқларни тўлади, уйни таъмирлади, кўп меҳнат қилди ва 2010 йилда вафот эди. Ундан кейин уйда ўғли яшашда давом этди, у ҳам солиқларни тўлади, коммунал хизматлар ҳақини ҳам, қўшимча хоналар қурди ва бошқа ишларни амалга оширди. 2019 йилга келиб ўғли уйга эгалик қилиш ҳуқуқни олиш учун судга мурожаат қилди. Шунда отасининг эгалик қилган даври ўғлининг эгалик қилган даврига қўшиб ҳисобланади ва ўғлида эгалик қилиш муддати пайдо бўлади. Бунинг учун фуқаролик судига эгалик ҳуқуқини этироф этиш ҳақида ариза билан мурожаат қилиш лозим. ✅ Шунга кўра сиз ҳам юқоридаги асосларга биноан судга даъво қилишингиз керак деб ҳисоблаймиз.
Show more ...
44 423
591
3 курс талабаси МТТда психолог лавозимига ишга олиниши мумкинми? 4479-савол: Assalomu alaykum. Men O'zbekiston Milliy Universitetining Ijtimoiy fanlar fakulteti Amaliy psixologiya yo'nalishi sirtqi talimda 3kurs tugatib 4 kursga o'tdim. Men maktab,  MTT da yoki boshqa tashkilotda psixolog bo'lib ishlasam bo'ladimi va nechchi stavkada ishlashim mumkin. Javob uchun rahmat ЮРИСТ ЖАВОБИ: Вазирлар Маҳкамасининг 2018-йил 5-январдаги 10-сон қарорига 1-иловада тасдиқланган Низомга илова 3-бандига асосан тил ўқитувчилари, ўқитувчи-дефектолог, ўқитувчи-логопед, психологлар, олий маълумотли тайёргарлик (5 ёшдан 6-7 ёшгача) гуруҳи тарбиячилари лавозимига олий таълим муассасаларининг педагогика йўналишларида кундузги, кечки, сиртқи бўлимларнинг 3-4-курсларида ўқиётган шахслар ишга қабул қилиниши мумкинлиги белгиланган. ✅ Шунга кўра, сиз ҳам 3-курсда ўқиётган бўлсангиз, МТТда психолог лавозимига ишга қабул қилинишингиз мумкин. Каналимизга уланинг                 👇👇 👉 ‌‌
Show more ...
Юрист Хизмати Қабул
Ўзингизни қийнаётган барча саҳаларга оид муаммоли саволларингизни юборишингиз мумкин.
50 309
530
❗️Эртага — барча учун иш куни! 2023 йил 22 декабрдаги ПФ-213-сонли Президент фармонига асосан, 5 кунлик иш ҳафтасида ишлайдиган ходимлар учун 13 апрель, шанбадаги дам олиш куни 12 апрель, жумага кўчирилган. Яъни 5 кунликлар бугун дам оляпти, аммо эртага ишга Шундай қилиб, эртага, 13 апрель — шанба куни БАРЧҲА ходимлар учун ИШ КУНИ ҳисобланади. 6 кунликлар ҳам, 5 кунликлар ҳам ишга чиқади.
Xushnudbek.uz
Huquqshunos Xushnudbek Xudoyberdiyevning Telegram kanaliga xush kelibsiz! •youtube.com/c/xushnudbekxudoyberdiyev •fb.com/xushnudjohn •twitter.com/xushnudbeck •instagram.com/xushnudbek •tiktok.com/@xushnudbekofficial Reklama uchun: @xushnudbekreklama
73 756
647
Иссиқхона қуриш учун берилган ерга турар ё нотурар бино қурса бўладими? 4478-савол: Ассалому алейкум! Мен ер участкаси сотиб олганман. Ер участкасини кадастр хужжати "Гулчиликка ихтисослашган иссикхона ташкил этиш учун"  деб берилган. Бу ерни якка тартибда уй куриш учун нотурар жой дан турар жой га утказиб буладими? ЮРИСТ ЖАВОБИ:Мумкин эмас деб ҳисоблаймиз. ❗️Чунки ер қандай мақсадларга берилган бўлса, ундан фақат шу мақсадда фойдаланиш лозим бўлади. Ер кодексининг 2-моддасига кўра ерлардан оқилона, самарали ва белгиланган мақсадда фойдаланишни таъминлаш ер тўғрисидаги қонунчиликнинг асосий принсипларидан бири ҳисобланади. Бунда кодекс 9-моддасига асосан ерлар асосий фойдаланиш мақсадига қараб ер фонди тоифаларига бўлинади. Сизлардаги ҳолатда ҳам ер участкаси қишлоқ хўжалигига мўлжалланган ерлар тоифасига кириши мумкин. Чунки иссиқхоналар барпо этиш қишлоқ хўжалигига мўлжалланган ер участкаси фондидан ажратилади. Ушбу кодекс 44-моддасида суғориладиган ерларни қишлоқ хўжалигидан бошқа мақсадлар учун, шу жумладан саноат ва фуқаролик обектлари (бинолар ва иншоотлар) қурилиши учун ажратилмаслиги белгиланган. ❗️Агар сизлар бу тоифадаги ер участкасида турар ёки нотурар бино қурсангиз, унда кодекснинг 36-моддасига биноан ер участкага бўлган ҳуқуқ бекор қилинади. Каналимизга уланинг                 👇👇 👉 ‌‌
Show more ...
Юрист Хизмати Қабул
Ўзингизни қийнаётган барча саҳаларга оид муаммоли саволларингизни юборишингиз мумкин.
49 824
30
❗️Мактабларда ўқитувчилар ўртасида навбатчилик шакллантирилиши қонунийми? 4412-савол: Assalomu aleykum meni murojatim shundan iboratki man maktabda fan o‘qituvchisi bo‘lib ishlab kelmoqdaman maktada dars kamligi un boshqa maktabda xam o‘rindoshlik vazfasida ishlab ro‘zg‘or tebratyapman shu ishlar ichida asosiy maktabimda navbatchilik qilish muammosi juda ko‘p bo‘lyapti men turolmayman vaqtim yo‘q desam bu majburiy adliyadan tasdiqlatib keldik deyishdi 206 - modda bekor qilingan deb aytishyapti xam bu rostan xam majburiymi shuni qonunniy isbotlashimga amaliy yordam berila javob un oldindan raxmat ЮРИСТ ЖАВОБИ:Ўқитувчиларга навбатчилик қилиш мажбуриятини юклашнинг ўзи нотўғри. Чунки Мактабгача ва мактаб таълими вазирининг 2023-йил 20 - декабрдаги 565-сонли буйруғига 37-иловада тасдиқланган Умумий ўрта таълим муассасалари (айрим фанлар чуқур ўрганиладиган ихтисослаштирилган муассасалар) ходимларининг лавозим мажбуриятларига асосан ўқитувчининг лавозим мажбуриятлари ичида бундай мажбурият йўқ. 🚫 Демак, бу ҳолатда иш берувчи ходимга бу мажбуриятни юклаши ҳуқуқий жиҳатдан асосга эга эмас. Меҳнат кодекси кодекс 115-моддасига кўра иш берувчи ходимдан қуйидагиларни талаб қилишга ҳақли эмас: 🚫 агар ушбу Кодексда ва бошқа қонунда бошқача қоидалар белгиланмаган бўлса, меҳнат шартномасида шарт қилиб кўрсатилмаган ишни бажаришни; 🚫 қонунга хилоф бўлган, ходимнинг ёки бошқа шахсларнинг ҳуқуқларини бузадиган, ҳаёти ва соғлиғини хавф остига қўядиган, уларнинг шаъни, қадр-қиммати ҳамда ишчанлик обрўсига путур етказадиган ҳаракатларни амалга оширишни. ❌ Бундан кўриниб турибдики, иш берувчи ўқитувчиларга уларнинг меҳнат вазифангиздан бошқа ишни буюриши қонунан таъқиқланади. ❗️Бундай вазиятларда иш берувчига нисбатан Маъмурий жавобгарлик тўғрисидаги кодекс 49-моддаси билан (яъни мансабдор шахс томонидан меҳнат ва меҳнатни муҳофаза қилиш тўғрисидаги қонун ҳужжатларини бузиш) базавий ҳисоблаш миқдорининг беш бараваридан ўн бараваригача миқдорда жарима солишга сабаб бўлади. ❌ Шунга кўра мактабда ўқитувчилар ўртасида навбатчиликни шакллантириш ноқонуний ҳисобланади. Каналимизга уланинг                 👇👇 👉 ‌‌
Show more ...
64 519
858
Ерни қарори йўқ бўлса, фермер олиб қўйиши мумкинми? 4476-савол: Bizning yer uchastkamizni qarori yuq. 3 yil oldin uy joyga beton quyilgan va 3 yillik yer soligi tolangan. Yer kadastiri bor, faqat qaroro yuq. Hozirgi paytda bir fermer mening yerim deb sudga bergan.Sud qarori xech narsani tekshirmay buzilsin deb qaror chiqargan. Bu vaziyatni qonunlar boyicha korib yordam berishiningizni sorayman. ЮРИСТ ЖАВОБИ: Кўчмас мулкка бўлган ҳуқуқларни давлат рўйхатидан ўтказиш тўғрисидаги қонун 11-моддасига биноан Биноларга, иншоотларга, шунингдек қурилиши тугалланмаган обектларга бўлган ҳуқуқларни давлат рўйхатидан ўтказиш ушбу кўчмас мулк жойлашган ер участкасига бўлган ҳуқуқлар давлат рўйхатидан ўтказилмагунига қадар амалга оширилмайди. ☝️Мана шу қоидага кўра сиздаги вазиятда ер учаскасига нисбатан сиздаги вазиятда ҳуқуқ давлат рўйхатидан ўтмаганлиги учун сиз унинг устидаги ҳар қандай қурилмани давлат рўйхатидан ўтказа олмайсиз. ❗️Сиздаги кадастр маълумоти ҳуқуқни белгиловчи ҳужжат эмас. Қаранг! Қонуннинг 31-моддасига асосан 2021-йилнинг 1-августидан эътиборан қабул қилинган ер участкасига бўлган ҳуқуқларни белгиловчи ҳужжатлар қуйидагилардан иборат: ➖ ер участкасига бўлган мулк ҳуқуқини давлат рўйхатидан ўтказиш учун — давлат мулкини бошқариш бўйича ваколатли давлат органлари томонидан бериладиган давлат ордери; ➖ 1998-йилнинг 1-июлидан 2021-йилнинг 1-августига қадар қабул қилинган ер участкасига бўлган ҳуқуқларни белгиловчи ҳужжатлар қуйидагилардан иборат: ➖ ер участкасига мерос қилиб қолдириладиган умрбод эгалик қилиш ҳуқуқини давлат рўйхатидан ўтказиш учун — маҳаллий давлат ҳокимияти органларининг ер участкасини бериш ҳақида ўз ваколатлари доирасида қабул қилинган қарорлари, нотариал тасдиқланган мерос қилиб олиш ҳуқуқи тўғрисидаги гувоҳнома, ушбу ҳуқуқнинг аукционда сотиб олинганлигига асосан туман (шаҳар) ҳокими томонидан берилган давлат ордери асос бўлади. Шу қоидага кўра сизлардаги вазиятда ер участкасига нисбатан ушбу ҳуқуқларни тасдиқловчи ҳужжатлар мавжуд бўлмаса, бу ер сизларга тегишли бўлмайди. Агар ер участкаси қишлоқ хўжалигига мўлжалланган ер участкаси тоифасига кирса, сизлар унга нисбатан ҳуқуқни давлат рўйхатидан ўтказа олмайсизлар.  Чунки бундай ер участкаси фермер ёки деҳқон хўжалигига берилади. ✅ Агар бу ер мазкур тоифага кирмаса, унда аукцион орқали уни сотиб олишингиз мумкин. Бунинг учун ҳудудий кадастр бўлимига мурожаат қилиш керак. Каналимизга уланинг                 👇👇 👉 ‌‌
Show more ...
Юрист Хизмати Қабул
Ўзингизни қийнаётган барча саҳаларга оид муаммоли саволларингизни юборишингиз мумкин.
54 968
237
🌙 Мусулмон аҳлига Рамазон ҳайити муборак бўлсин! 👉

owOHMDIeKgGuAaAFQtYICNBUOgLZjKiIsIJSef.mp4

68 014
940
🌙 Рамазон Ҳайити муборак бўлсин! 👉
75 433
1 269
❗️*Муборак Рамазон ҳайити муносабати билан неча кун дам оламиз?* Рамазон ҳайити арафасида қуйидаги *кунлар дам олинади:* 🟢 10 апрель (чоршанба) Рамазон ҳайити байрами; 🟢 11 апрель (пайшанба) барча ходимлар учун дам олиш куни (иш ҳафтасининг туридан қатъи назар) 🟢 12 апрель (жума) 6 кунлик иш ҳафтасида ишлайдиган ходимлар учун қўшимча дам олиш куни,  5 кунлик иш ҳафтасида ишлайдиган ходимлар учун 13 апрель куни шанбадан кўчирилган дам 🔴 13 апрель – (шанба)  барча учун иш куни; 🟢 14 апрель – (якшанба)  дам олиш куни. 👉
Show more ...
Huquqiy axborot
Қонунчиликдаги янгиликлар, норматив ҳуқуқий ҳужжатлардаги фойдали маълумотлар ва ҳуқуқий ислоҳотлар ҳақида тезкор маълумотлар тақдим этувчи канал. ▪️|Лотин алифбосида — @huquqiyaxborot_lotin ▪️|Рус тилида — @pravoinf ▪️|Инглиз тилида — @leginf
52 887
154
2024 йил солиғини тўлиқ тўлашни талаб қилиш тўғрими? 4475-савол: Ассалому алайкум хурматли юрист. Мен туман мактабида ўқитувчи булиб ишлайман мактаб директори ва  района бошлиги мени 2024 йил учун солигимни хаммасини тулашга мажбурлаяпти ши тугрими. Солигни ойма ой тулаб боряпман лекин Бу кам дейишди шу тугрими жавоб учун олдиндан рахмат. ЮРИСТ ЖАВОБИ: 🙅‍♂ Умумий қоида бўйича ходимнинг иш ҳақидан унинг розилигисиз ушлаб қолиш мумкин эмас. ❗️Бироқ бунга истесноли ҳолатлар ҳам мавжуд. Хусусан Меҳнат кодексининг 269-моддасига мувофиқ ходимнинг ёзма розилигидан қатъи назар солиқлар ва йиғимларни ундириш учун унинг иш ҳақидан ушлаб қолиш мумкинлиги белгиланган. 💁‍♂ Соддароқ айтганда, иш берувчи ходимнинг розилигисиз унинг иш ҳақидан солиқларни ёки бмтларни ушлаб қолиши Солиқ кодексининг 386-моддасига асосланган ҳолда солиқни ҳисоблаб чиқариш, ушлаб қолиш ва буджетга ўтказишни амалга оширади. ❓Мана шу жойда савол туғилади. Иш берувчи солиқ агенти сифатида ходимдан ҳар қандай солиқни ушлаб қолиши мумкинми? Йўқ ундай эмас, Солиқ кодексининг 386- 387-моддалари мазмунига мувофиқ солиқ агенти (иш берувчи) фақат солиқ тўловчи бўлган ходимдан солиқларни ушлаб қолишга ваколати саналади. ☝️Эътибор беринг! Фақат “солиқ тўловчи”дан ушлаб қолади. Ана энди масалани янада аниқлаштирамиз. Ходим солиқ тўловчи мақомига қачон эга бўлади. 1⃣ Биринчидан, ходим Солиқ кодексининг 364-моддасига кўра жисмоний шахслардан олинадиган даромад солиғининг солиқ тўловчиси саналади. Шунга асосан иш берувчи ходимдан СК 387-моддасини асос қилиб тўланган иш ҳақидан даромад солиғини ушлаб қолишга ваколатлидир. ❗️Шунга кўра иш берувчи солиқ агенти сифатида ходимга тўлаган иш ҳақи миқдоридан даромад солиғи суммасини ушлаб қолиши лозим. 2⃣ Иккинчидан, ходимнинг номида мол-мулк ва ер бўладиган бўлса, унда ҳам юқоридаги тартибда солиқларни ушлаб қолиши ёки ходимдан солиқларни ўз вақтида тўлашни талаб қилиши мумкин. Мол-мулк ва ер солиғини тўлаш муддати йилига 2 марта 1 апрель ҳамда 15 октабрь этиб белгиланган. ❌ Шунга кўра сиздан солиқни бирдагина тўлиқ тўлашни талаб қилиш мумкин эмас. Каналимизга уланинг                 👇👇 👉 ‌‌
Show more ...
Юрист Хизмати Қабул
Ўзингизни қийнаётган барча саҳаларга оид муаммоли саволларингизни юборишингиз мумкин.
49 230
203
Ҳайит куни рўза тутиш ҳақида ❓Ҳайит куни тонгда саҳарлик қилиб рўза тутишни ният қилиб, ҳайит намозидан сўнг оғиз очиладими? Устоз Эркинжон домла Гулмуродов 👉

IMG_6885.MOV

64 446
894
🌙 Рамазон ҳайити намози қандай ўқилади? Яқинларга улашинг, бир эслаб олайлик 👉

Namoz o'qilish tartibi_tg.mp4

39 180
189
⚡️Ҳайит куни метро ҳам бепул бўлади Ҳукумат ташаббуси билан "Рамазон ҳайити" муносабати билан 10 апрель санасида Тошкент метроси соат 04:00 дан 10:00 га қадар йўловчиларга бепул хизмат кўрсатади. “Тошкент метрополитени” ДУК
37 247
61
⚡️Бугун барча ходимлар 1 соат вақтли кетишга қонуний ҳаққи бор. Меҳнат кодекси 187-моддасига кўра, байрам (ишланмайдиган) кунлари арафасида кундалик иш (смена) муддати барча ходимлар учун камида бир соатга қисқартирилади. Эртага 10 апрель - Рамазон ҳайити байрами. Шунга кўра Ҳайит байрамида ходимлар кетма кет 3 кун(10-12 апрель) дам олади. Каналимизга уланинг                 👇👇 👉
Юрист Хизмати | Расмий канал
Каналимиз № 0005006503 - сонли давлат рўйхатидан ўтган гувоҳнома асосида фаолият юритади. Саволларингизни каналимиз постларида кўрсатилган мурожаат ботларимиз орқали юборишингиз мумкин. Реклама бўйича: t.me/Yurist_Xizmati_reklama
42 861
181
10 апрель Рамазон ҳайити байрами муносабати билан *республикадаги барча* жамоат транспорти воситалари* соат 04:00 дан 10:00 га қадар *аҳолига* бепул* хизмат кўрсатади. 
62 599
467
🌙 Рамазон ҳайити намозининг ўқилиш вақтлари эълон қилинди
55 115
345
Янги қурилган кўп қаватли уйларнинг подвалини шахсийлаштирса бўладими? 4474-савол: Ассалому алейкум, ман янги қурилган кўп қаватли уйнинг биринчи қаватидан квартира олганман, квартирамнинг хар битта хонасини тагида подваллар мавжуд, мазкур подвалларга кадастр хужжати қилиб биламанми ўзимнинг номимга? Ёки бошқа қонуний йўллар борми подвални ўзимнинг номимга қилиб олиш учун? Ёки мазкур подвалларни ўзбошимчалик билан эгаллаб ишлатаверсам хам бўладими? Рахмат. ЮРИСТ ЖАВОБИ: ❌ Подвал ўша кўп қаватли уйда яшовчиларнинг умумий мулки, уни ўз номингизга ўтказа олмайсиз деб ҳисоблаймиз. Фуқаролик кодексининг Кўп квартирали уйдаги турар жойлар ва яшаш учун мўлжалланмаган жойлар мулкдорларига шу уйнинг умумий жойларини, хусусан даҳлизлар, техник қаватлар, ертўлалар умумий мол-мулк улушли мулк ҳуқуқи асосида тегишли бўлади. ❗️Шунга кўра сиздаги ҳолатда подвал ўша уйда яшовчиларнинг умумий мулкидир. Кўп квартирали уйларни бошқариш тўғрисидаги қонун 13-моддаси биринчи қисмига асосан умумий мол-мулк жойларнинг мулкдорларига умумий улушли мулк ҳуқуқи асосида тегишли бўлади. ☝️Ушбу модданинг олтинчи қисмида умумий мол-мулкнинг айрим қисмлари умумий улушли мулкнинг барча иштирокчилари томонидан бир овоздан қабул қилинган қарор асосида бошқа шахсга берилиши мумкинлиги белгиланган. ✅ Демак, уйда яшовчи барча мулкдорларнинг розилигини олган тақдирдагина подвални ўз номингизга ўтказишингиз мумкин бўлади. Акс ҳолда у ерни эгаллашингиз ноқонуний ҳисобланади. Каналимизга уланинг                 👇👇 👉 ‌‌
Show more ...
Юрист Хизмати Қабул
Ўзингизни қийнаётган барча саҳаларга оид муаммоли саволларингизни юборишингиз мумкин.
35 833
141
⚡️Ходимлар Рамазон ҳайити муносабати билан 3 кун кетма-кет дам олишади Байрам 10 апрель кунига тўғри келиши муносабати билан, дам олиш кунлари кетма-кетлиги : 👉 10 апрель, чоршанба — Рамазон ҳайити куни; 👉 11 апрель, пайшанба — барча учун қўшимча дам олиш куни; 👉 12 апрель, жума — 13 апрел, шанба кунидан кўчирилган дам олиш куни. Шунингдек, 👉 13 апрель, шанба — барча учун иш куни; 👉 14 апрель, якшанба — барча учун дам олиш куни бўлади.
Рамазон ҳайити 10 апрел куни нишонланади
Ўзбекистон мусулмонлари идораси мамлакатда жорий йилда Рамазон ҳайити 10 апрел, чоршанба кунига тўғри келиши ҳақида қарор қабул қилди.
90 963
1 411
⚡️Рамазон ҳайити 2024 йил 10 апрель, чоршанба кунига тўғри келиши кутилаётганини маълум қилинди - Ўзбекистон мусулмонлари идораси Уламолар Кенгашининг навбатдаги онлайн мажлиси бўлиб ўтди. Унда Рамазон ҳайитини кўтаринки руҳда ўтказишга оид кун тартибидаги бир қатор масалалар муҳокама қилинди. Дунёдаги йирик фатво идоралари ва нуфузли астрономия институтлари ахборотлари батафсил кўриб чиқилди. Туркий давлатлар диний идоралари масъуллари билан ҳам ушбу масала кенг муҳокама қилинди. Ўзбекистон мусулмонлари идораси Уламолар Кенгаши фиқҳий қоидалар, фалакий ҳисоб-китоблар ва астрономик кузатишларга таянган ҳолда жорий йилда диёримизда Рамазон ҳайити 2024 йил 10 апрель, чоршанба кунига тўғри келиши кутилаётганини маълум қилди.
Show more ...
Muslim.uz
🌙 РАМАЗОН ҲАЙИТИ ТЎҒРИСИДА 🌐Ўзбекистон мусулмонлари идораси Уламолар Кенгашининг навбатдаги онлайн мажлиси бўлиб ўтди. Унда Рамазон ҳайитини кўтаринки руҳда ўтказишга оид кун тартибидаги бир қатор масалалар муҳокама қилинди. 💬Йиғилишда уламолар ва соҳа мутахассислари томонидан Рамазон ҳайити қайси кунга тўғри келиши ва ҳижрий 1445 йил Шаввол ойи ҳилолини кузатиш бўйича қилинаётган ишлар ҳамда фалакий ҳисоб-китоблар атрофлича таҳлил қилинди. 🔭 Шунингдек, дунёдаги йирик фатво идоралари ва нуфузли астрономия институтлари ахборотлари батафсил кўриб чиқилди. Туркий давлатлар диний идоралари масъуллари билан ҳам ушбу масала кенг муҳокама қилинди. 🗣Ўзбекистон мусулмонлари идораси Уламолар Кенгаши фиқҳий қоидалар, фалакий ҳисоб-китоблар ва астрономик кузатишларга таянган ҳолда жорий йилда диёримизда Рамазон ҳайити 2024 йил 10 апрель, чоршанба кунига тўғри келиши кутилаётганини маълум қилди. Ўзбекистон мусулмонлари идораси Матбуот хизмати #Ramazon #hayit MUSLIM.UZ GA 😎 OBUNA BO'LING ✅ VA ULASHING🗣 ▶️ YouTube | 📷Instagram | 🔵 Facebook
35 365
116
Директор ходимни "ўрта махсус"сан деб ишдан бўшатиши мумкинми? 4473-савол: Ассалому алайкум. Хурматли Юрист ХИЗМАТИ жамоаси Мактабда ишлаганимга Сал Кам 30 йил бу‌лди, у‌рта махсусман. Пенсияга 6 йил колди. Директорим Олий таълимга у‌кишга кирмасангиз шартномани бекор киламан деяпти,шу ту‌грими? Жавоб учун олдиндан Рахмат ЮРИСТ ЖАВОБИ: Мактабгача ва мактаб таълими вазирининг 16.08.2023 йилдаги 247- сонли буйруғига кўра мактабга педагог ходимларни ишга қабул қилишда қуйидаги мезонларга риоя қилиш лозим: 👉 Бошланғич синф ўқитувчиси лавозимига - бошланғич таълим ихтисослиги бўйича камида ўрта махсус педагогик маълумот ёки камида бакалавр даражасидаги олий маълумот ва бошланғич таълим ихтисослиги бўйича дарс бериш ҳуқуқи билан касбий қайта тайёрлов курсини тугатган мутахассислар; 👉 Фан ўқитувчиси лавозимига - фан йўналишига тўғри келувчи камида бакалавр даражасидаги олий педагогик маълумот ёки камида бакалавр даражасидаги олий маълумот ва фан йўналишига тўғри келувчи педагогик фаолият юритиш ҳуқуқи билан қайта тайёрлов курсини тугатган мутахассислар. Шунингдек, “Умумтаълим муассасаларида синфларни комплектлаш ҳамда тарификация рўйхатларини шакллантириш тартиби тўғрисида”ги низом (рўйхат рақами 3271) 15-бандига кўра ўрта махсус маълумотли педагог ходимларга ҳам тарификация асосида дарс бериш мумкин. ❗️Шунга кўра директор сизни ўрта махсус маълумотли эканлигингиз туфайли ишдан бўшата олмайди. Лекин дарс тақсимотида кам дарс тегиши мумкин. Каналимизга уланинг                 👇👇 👉 ‌‌
Show more ...
Юрист Хизмати Қабул
Ўзингизни қийнаётган барча саҳаларга оид муаммоли саволларингизни юборишингиз мумкин.
38 827
624
❗️Қашқадарёдаги мактабда бола тенгдошлари томонидан калтакланди Натижада боланинг умуртқаси синган. Куни кеча ижтимоий тармоқларда боланинг онаси ўғли соғайиб кетмаганини айтган мурожаат ҳам . Вилояти Мактабгача ва мактаб таълими бошқармаси ҳолат ўрганилиб, мактаб директори ишдан бўшатилганини маълум қилди. Таъкидланишича, ҳуқуқни муҳофаза қилувчи орган ходимлари ўқувчилар ўртасидаги жанжални ўрганган ва қонуний тўхтамга келинган.

IMG_6674.MP4

33 739
121
🇺🇿🇰🇿 Энди Ўзбекистон фуқаролари Қозоғистонда 30 кунгача рўйхатдан ўтмасдан қолиши мумкин Президент Тоқаев ўзаро саёҳат шартлари тўғрисидаги қонунни имзолади. Қозоғистонликлар ҳам худди шу муддатга Ўзбекистонда вақтинчалик рўйхатдан ўтмасдан қолиши мумкин.
Sputnik Uzbekistan | Янгиликлар
sputniknews.uz сайтининг расмий Telegram канали Ўзбекистон ва жаҳоннинг тезкор янгиликлари Лотинда 👉 @Sputnik_lotin Русская версия👉 @sputnikuzbekistan Фото/видеолар, ҳамкорлик таклифлари юбориш учун👉 @SputnikUz_feedbackbot
31 797
104
⚡️Чет элга ишлашга борувчиларнинг имтиҳон, виза ва йўл чиптаси харажатлари қоплаб берилади Президент фармонида хорижга вақтинча меҳнат фаолиятини олиб бориш учун жўнаб кетаётган ёки хорижда вақтинча ишлаётган фуқароларга тиббий, ижтимоий, ҳуқуқий, консуллик хизматларини кўрсатиш масалалари Хусусан, 2024 йил 1 июндан бошлаб «Хорижда иш» меҳнат миграцияси электрон дастурий мажмуасида рўйхатдан ўтган ва ташкиллаштирилган меҳнат миграциясига кетаётган фуқароларга: 👉 хорижий тиллар ва касб бўйича малака имтиҳонларини топшириш билан боғлиқ харажатларини қоплаш учун БҲМнинг 3 бараваригача;  👉 «Ишчи виза»ни расмийлаштириш билан боғлиқ харажатлар учун БҲМнинг 5 бараваригача; 👉 Йўл чиптаси учун БҲМнинг 2 бараваригача миқдорда компенсация тўланади.
Show more ...
Чет элга ишлашга борувчиларнинг имтиҳон, виза ва йўл чиптаси харажатлари қоплаб берилади
Президент фармонида хорижга вақтинча меҳнат фаолиятини олиб бориш учун жўнаб кетаётган ёки хорижда вақтинча ишлаётган фуқароларга тиббий, ижтимоий, ҳуқуқий, консуллик хизматларини кўрсатиш масалалари кўрсатиб ўтилган.
34 179
214
Имзони сохталаштирганлик учун қонунчиликда қандай жазо бор? 4472-савол: Assalomu alaykum meni bir tanishim boxchada ishlaydi boxcha asosiy edi keyin u o‘rindosh qlish maqsadida ariza yozib mudirani o‘rnidan imzo qo‘ygan va adliyadan aylantirgan endi mudira bilib qolib arizangni yoz bo‘lmasam jn kodeksning 288 moddasi bilan iah qo‘zg‘atama va 50 aklat jarima va 5 yilgacha ishlolmiysan va maktabdagi shartomangxam bekor bo‘lib ketadi va o‘sha arizaning asl nusxasini ber dyapti bo‘lmasa sudga beraman dyapti shunga nima qlsa bo‘ladi qanday yo‘l tutsa bo‘ladi arizaning asl nusxasi yuq  shunga yordam bersangiz oldindan raxmat ЮРИСТ ЖАВОБИ: Имзосини сохталаштирган бўлса, унинг бу ҳаракатларига нисбатан Жиноят кодекси 228-моддаси билан жиноят иши очиш мумкинлигини билдиради. Ушбу моддага кўра сохталаштирувчининг ўзи ёки бошқа шахс фойдаланиши мақсадида муайян ҳуқуқ берадиган ёки муайян мажбуриятдан озод этадиган расмий ҳужжатлар тайёрлаши ёки расмий ҳужжатларни қалбакилаштириши ёхуд бундай ҳужжатларни сотиши, шундай мақсадларда корхона, муассаса ёки ташкилотнинг қалбаки штамплари, муҳрлари, бланкаларини тайёрлаш ёхуд сотиш: ❗️базавий ҳисоблаш миқдорининг 50 бараваридан 100 бараваригача миқдорда жарима ёки уч юз олтмиш соатгача мажбурий жамоат ишлари ёхуд уч йилгача ахлоқ тузатиш ишлари билан жазоланади. ✅ Бу вазиятда ўзаро келишув билангина масалани ҳал қилиш мумкин. Ёки ҳақиқатда ёзилган ариза нусхасини топиш талаб этилади. Ҳақиқатда бошқа йўли йўқ. Каналимизга уланинг                 👇👇 👉 ‌‌
Show more ...
Юрист Хизмати Қабул
Ўзингизни қийнаётган барча саҳаларга оид муаммоли саволларингизни юборишингиз мумкин.
39 392
353
Қайнотани номида машина бор деб боллар пули бекор қилиниши тўғрими? 4471-савол: Meni nomimdan hech narsa yoʻq. Dadamni nomida 2009 Lasetti moshina bor, tomorqamiz 22 sotiq. Uyda uchta qoramol bor, onam 600000 pensiya oladi, men 300000 oylik olaman xotinim ishlamaydi 1 ta 2 yoshga bitta 1 yoshga toʻlgan bolam bor. Bizga bola nafaqasi chiqadimi onam xujjat olib borsa mahallada oʻz boshimchalik bilan olmayapti ekan ЮРИСТ ЖАВОБИ: Вазирлар Маҳкамасининг 2021-йил 21-октабрдаги 654-сон қарорига 1-илова билан тасдиқланган “Кам таъминланган оилаларни “Ижтимоий ҳимоя ягона реестри” ахборот тизими орқали аниқлаш, уларга кам таъминланган оилалар болалари учун нафақа ва моддий ёрдам тайинлаш ва тўлаш тартиби тўғрисида”ги низомга кўра оила: •  2022-йилдан бошлаб — оиланинг ҳар бир аъзосига жами бир ойлик ўртача даромад минимал истеъмол харажатлари миқдоридан ошмаганда кам таъминланган деб эътироф этилади. ☝️Юқоридаги қоиданинг мазмунига кўра оилангизнинг даромади ҳозир ҳар бир оила аъзосига ҳисоблаганда 621 минг сўмдан кам бўлса, унда сиз кам таъминланган деб топилишингиз мумкин. Бунда ушбу низом 22-бандига кўра оиланинг бир аъзосига тўғри келадиган бир ойлик ўртача даромад болалар нафақаси ёки моддий ёрдам олиш учун мурожаат қилинган ойдан олдинги ойгача бўлган охирги уч ойи учун ҳисоблаб чиқилган барча оила аъзоларининг даромади суммасини оила таркибига киритиладиган оила аъзолари сонига ва учга бўлиш орқали аниқланади. Қаранг! Мазкур низомнинг 20-бандига кўра ариза берувчи оиласининг ҳар бир аъзосига тўғри келадиган бир ойлик ўртача жами даромад ушбу Низомнинг 7-бандида белгиланган мезондан ошганда болалар нафақаси ёки моддий ёрдам тайинлаш рад этилади. Шунингдек, мазкур бандга кўра ариза берувчи ёки унинг оила аъзолари фойдаланувида бўлган ёхуд ишончномага асосан ишлаб чиқарилган муддати 12 (оила аъзоси I ёки II гуруҳ ва 18 ёшгача болаликдан ногирон бўлса 7) йилдан ошмаган, техник ҳолати соз бўлган енгил автотранспорт воситаси бўлганда ҳам ариза рад қилинади. Энди қаранг! ушбу нормада оила аъзолари жумласи қўлланилган. Мазкур низом 22-бандига кўра болалар нафақаси ёки моддий ёрдам олиш учун оила таркибига: 👉 фуқаролик ҳолати далолатномаларини ёзиш органларида никоҳи қайд этилган эр-хотин (ёки уларнинг бири) ва улар билан бирга яшайдиган ва уларнинг қарамоғида бўлган болалар (шу жумладан, васийликка (ҳомийликка) олинган болалар); •  шунингдек, ота-оналари билан бирга яшайдиган ва ўз оиласига эга бўлмаган 18 ёшдан катта болалар (шу жумладан, васийликка (ҳомийликка) олинган болалар), ёлғиз оналар (оталар) киритилади. 💁‍♂ Бунда низомнинг 22-бандига кўра болалар нафақаси ёки моддий ёрдам олиш учун оила таркибига фуқаролик ҳолати далолатномаларини ёзиш органларида никоҳи қайд этилган эр-хотин (ёки уларнинг бири) ва улар билан бирга яшайдиган ва уларнинг қарамоғида бўлган болалар (шу жумладан, васийликка (ҳомийликка) олинган болалар), шунингдек, ота-оналари билан бирга яшайдиган ва ўз оиласига эга бўлмаган 18 ёшдан катта болалар (шу жумладан, васийликка (ҳомийликка) олинган болалар), ёлғиз оналар (оталар) киритилади. ❗️Шунга кўра сиздаги вазиятда ота-онангиз, сизнинг оила аъзойингиз таркибига кирмайди деб ҳисоблаймиз. Бу вазиятда туман адлия бўлимига мурожаат қилишингиз лозим. Каналимизга уланинг                 👇👇 👉 ‌‌
Show more ...
43 001
181
⚡️Вилоят миқёсида "Йилнинг энг энг яхши мактаб жамоаси - 2023" танлови 1, 2 ва 3-ўрин ғолиблари учун махсус сертификатлар ғолиб мактаб жамоаларига етказилади. Бунинг учун ушбу рўйхатдаги ғолиб бўлган ҳар бир мактаблардан бир масъул ходим 'га боғланишларини ва телефон номер қолдиришларини сўраймиз. Эътибор беринг! Ҳар бир мактабдан фақат бир маъул ходим бизга боғланиши керак. 👉
Юрист Хизмати | Расмий канал
⚡️Расман: "ЙИЛНИНГ ЭНГ ЯХШИ МАКТАБ ЖАМОАСИ — 2023" танловига старт берилди Юрист хизмати телеграм канали ўтган 2023 йилда йилнинг энг намунали, энг кучли ва энг яхши мактаб жамоасини аниқлаш ва уларни рағбатлантириш мақсадида ижтимоий лойиҳа сифатида "Йилнинг энг яхши мактаб жамоаси — 2023" танловига старт берилди. Ижтимоий сўровнома овоз бериш йўли орқали ўтказилади ва Республика миқёсида энг кўп овоз тўплаган 1, 2 ва 3-ўринларни эгаллаган мактаб жамоасига эсдалик совғалар, пул мукофотлари, диплом ва махсус сертификатлар, вилоятлар кесимида 1, 2 ва 3-ўринни эгаллаган мактаб жамоасига эса махсус сертификатлар топширилади. Овоз бериш @Yil_maktab_jamoasi_bot орқали амалга оширилади. Овоз бериш жараёни адолатли бўлишини таъминлаш мақсадида бир шахс фақат бир мартагина овоз берадиган махсус бот ишлаб чиқилди. Овоз бериш 2024 йил 29 февралга қадар давом этади. Танлов тафсилотлари @Yurist_Xizmati расмий каналимиз орқали ёритиб борилади. Мактаблар рўйхати👇 @Yil_maktab_jamoasi_bot
34 963
42
⚡️Республика бўйлаб педагог кадрлар аттестациясининг малака (тест) синовлари ўтказиш Каналимизга уланинг                 👇👇 👉 ‌‌
58 608
858
⚡️15 апрелдан бошлаб педагог кадрлар аттестациясининг малака тоифа (тест) синовлари бошланади ✅ Аввал хабар қилинганидек, мактабгача, умумтаълим ва мактабдан ташқари таълим муассасалари педагоглари 2024 йил 15 апрелдан бошлаб аттестатсиядан ўтказилади. ❗️ Мактабгача ва мактаб таълими вазирлиги томонидан мазкур малака тоифа синовлари ўтказилиш вақти белгиланди. Унга кўра: ➖Информатика ва ахборот технологиялари фанидан — 15-16 апрель кунлари (2 кун);Бошланғич таълим фанидан — 17-22 апрель кунлари (5 кун);Ўзбек тили (Давлат тили), Она тили ва адабиёт фанларидан — 23-25 апрель кунлари (3 кун);Математика фанидан — 26-29 апрель кунлари (3 кун);Физика, Кимё фанларидан — 30 апрель — 1 май кунлари (2 кун);Биология, География, Иқтисодий билим асослари фанларидан — 2-3 май кунлари (2 кун);Мусиқа маданияти, Тасвирий санъат ва чизмачилик, Технология (сервис хизмати йўналиши-қиз болалар учун), Технология (технология ва дизайн йўналиши - ўғил болалар учун) фанларидан — 4-6 май кунлари (2 кун);Рус тили (миллий мактабларда), Рус тили ва адабиёти (рус), Қорақалпоқ тили (Давлат тили) фанларидан — 7-8 май кунлари (2 кун);Француз тили, Ҳинд тили, Инглиз тили, Корейс тили, Немис тили, Қирғиз тили ва адабиёти, Қозоқ тили ва адабиёти, Тожик тили ва адабиёти, Туркман тили ва адабиёти, Хитой тили, Япон тили фанларидан — 10-13 май кунлари (3 кун);Давлат ва ҳуқуқ асослари, Тарбия, Тарих фанларидан — 14-15 май кунлари (2 кун);Чақирувга қадар бошланғич тайёргарлик, Жисмоний тарбия фанларидан — 16-17 май кунлари (2 кун);Мактабгача таълим муассасалари тарбиячиси, Мактабгача таълим ташкилоти психологи, Тарбиячи-методист, Ўқитувчи-дефектолог, Ўқитувчи-логопед, Мусиқа ходими (Мусиқа раҳбари), Психологларга — 18 май кунлари (1 кун);Қорақалпоқ тили ва адабиёти фанидан — 20-21 май кунлари (2 кун) малака тоифа синовлари ўтказилади. 👉 Маълумот учун: малака тоифа синовлари ўтказиладиган жой манзиллари қуйида келтириб ўтилган.
Show more ...
70 823
1 241
Гуруҳ раҳбарлиги учун қўшимча ҳақни давомадга боғлаб қўйиш тўғрими? 4470-савол: Ассалому алайкум хурматли юрист мен касб хунар мактабида ишлайман бизда гурух рахбарларига гуруҳининг давоматини фойизига қараб гурух рахбарлик пулни тўлашмоқда  шуки шу тўғрими жавоб учун олдиндан рахмат ЮРИСТ ЖАВОБИ:Гуруҳ раҳбарлиги учун қўшимча ҳақни давомадга боғлаб қўйиш нотўғри. Чунки қарорда бу ҳақида қоида белгиланмаган. Қаранг! Вазирлар Маҳкамасининг 2008-йил 1-августдаги 165-сон қарорига 4-иловада тасдиқланган Ўрта махсус, касб-ҳунар таълими муассасалари ходимлари меҳнатига ҳақ тўлаш тўғрисидаги Низом 10-бандига кўра ўрта махсус ёки касб-ҳунар таълими муассаси педагог ходимига базавий тариф ставкасига гуруҳ раҳбарлиги учун — меҳнатга ҳақ тўлашнинг энг кам миқдорининг 35,2 фоизи қўшимча тўлов тўланиши белгиланган. 💁‍♂ Бу дегани мазкур тўлов муассаса раҳбарининг буйруғи асосида гуруҳ раҳбари этиб тайинланган педагог ходимга гуруҳдаги ўқувчилар сонидан қатъий назар тўланади. ❗️Сизга ушбу вазиятда туман адлия бўлимига мурожаат қилишингизни тафсия қиламиз. Каналимизга уланинг                 👇👇 👉 ‌‌
Show more ...
36 150
111
Маҳалла раиси лавозимига кимлар сайланиши мумкин? 4469-савол: Кимлар махалла фукоралари йигини райси (оксоколи)этиб сайланиши мумкин Савол Махалла фукоралари йигини райси лавозимга кимлар у‌з номзодини куя олади, махалла фукоралари йигини райси сайланишига кадар вактинча бажарувчи ёки вактинча тайинлов кадар уч-ой синов муддатда Туман хокимнинг у‌ринбосари махалла ва ойлани куллаб -кувватлаш булим бошлиги тамонидан урта,урта махсус маьлумотга эга шахсни махалла фукоралари йигини райси лавозимга тайинланган Конунда урта,урта махсус махалла фукоралари йигини райси ишлаш мумкинми ёки мумкин эмасми конуний жавоб беришингиз сураймиз ЮРИСТ ЖАВОБИ: Фуқаролар йиғини раиси сайлови тўғрисидаги қонунга мувофиқ, раис(оқсоқол)ни сайлаш тартиби белгиланган. ☝️Ушбу қонуннинг 17-моддасига биноан, Фуқаролар йиғини раиси лавозимига номзодлар Ўзбекистон Республикаси фуқаролари бўлиши, қоида тариқасида, олий маълумотга эга бўлиши, бевосита сайловга қадар камида беш йил тегишли ҳудудда доимий яшаётган бўлиши, ташкилотчилик қобилиятига, давлат органларида ёки нодавлат нотижорат ташкилотларида ёхуд тадбиркорлик ва бошқа хўжалик фаолияти соҳасида иш тажрибасига, шунингдек ҳаётий тажрибага ва аҳоли ўртасида обрў-эътиборга эга бўлиши керак. ✅ Шундай экан, Олий маълумотли бўлиши императив норма эмас шунинг учун ўрта маълумотлилар ҳам раис этиб сайланиши мумкин. ❗️Бундан ташқари, Суд томонидан муомалага лаёқатсиз деб топилган, оғир ёхуд ўта оғир жиноятларни содир этганлик учун судланганлик ҳолати тугалланмаган ёки судланганлиги олиб ташланмаган шахслар, шунингдек суднинг ҳукми билан озодликдан маҳрум этиш жойларида сақланаётган, ҳақиқий ҳарбий хизматдаги шахслар, диний ташкилотларнинг профессионал хизматчилари фуқаролар йиғини раиси лавозимига сайланиш учун номзод этиб кўрсатилиши мумкин эмас.
Show more ...
50 865
384
🔴 Қизил чироқда турганингизда ҳам тез ёрдам машинасига йўл беринг! СИЗ ҲЕЧ ҚАНДАЙ ЖАРИМА ТЎЛАМАЙСИЗ!

file

44 678
176
Интизомий жазоси бор ходимга устама тўлаш мумкинми? 4468-савол: Ассалаума алейкум. Саволим бор эди. Тиббиет ходимларини рагбатландириш тугрисидаги низомда ойлик устамалар курсатилган. Саволим шуки, интизомий жазоси бор ходимга устама тулаш мумкинми? Шунга батафсил жавоб берсангиз ЮРИСТ ЖАВОБИ:Устама тўланади. Интизомий таъсир чораси қўлланилиши ходимнинг меҳнати билан боғлиқ рағбатлантириш чораларини қўлламаслик учун асос бўлмайди. Янги таҳрирдаги Меҳнат кодексининг 299-моддаси бешинчи қисмига мувофиқ интизомий жазонинг амал қилиш муддати ичида ходимга нисбатан рағбатлантириш чоралари, шу жумладан меҳнатга ҳақ тўлаш тизимига кирмайдиган ва меҳнат натижаларига асосланмаган мукофотлар ҳам (байрамлар, шу жумладан касб байрамлари, юбилейлар муносабати билан ва ҳоказолар) қўлланилмайди. ☝️Қоиданинг мазмунига кўра, интизомий таъсир чораси қўлланилган ходимларга меҳнат натижасига асосланмаган рағбатлантириш чоралари қўлланилмайди. Иккинчидан, ушбу модданинг тўринчи қисмида иш ҳақи, мукофотлар, қўшимча тўловлар, устамалар ва меҳнатга ҳақ тўлаш тизимида назарда тутилган бошқа тўловлар рағбатлантириш турлари жумласига кирмайди. ✅ Юқоридаги қоиданинг мазмунига кўра, сиз устамага бемалол даъвогарлик қилишингиз мумкин. Каналимизга уланинг                 👇👇 👉 ‌‌
Show more ...
Юрист Хизмати Қабул
Ўзингизни қийнаётган барча саҳаларга оид муаммоли саволларингизни юборишингиз мумкин.
41 370
383
Алиментдан воз кечиш эвазига уй олиш мумкинми? 4467-савол: Assalomu alaykum meni 4 ta farzandim bor turmush oʻrtogʻim boshqa ayol bilan yashamoqchi rasman ajrashmaganmiz u menga yashab turgan yumizni bermoqchi evaziga aliment toʻlashdan kechishini aytyabdi bu holatda men nima qilsam boʻladi .Men xech qayerda ishlamayman. ЮРИСТ ЖАВОБИ:Алиментдан воз кечиш эвазига уйни бериш мумкин эмас. ❗️Фуқаролик кодексининг 344-моддасига асосан алиментлар ундириш ҳақидаги талабни ҳисобга ўтказишга йўл қўйилмайди. Бу дегани, уйни бериб алиментдан озод қилишни сўраб бўлмайди. Агар уй-жой турмуш даврингизда орттирилган ва турмуш ўртоғингизнинг номида бўлса, унда Оила кодексининг 23-моддасига кўра бу уй сизларнинг умумий мулкингиздир. ОК 28-моддасига асосан уйнинг 50 фоизи шундоқ ҳам сизники бўлади. Агар никоҳдан олдин олинган ёки турмуш ўртоғингизга ҳадя ёҳуд мерос тарзида қолган бўлса, бу уй унинг шахсий мулки ҳисобланади. Ушбу вазиятда сизга уйнинг қийматидан келиб чиқиб ҳаракат қилишни тавсия қиламиз. ✅ Агар уйнинг қиймати юқори бўлиб, фарзандларингиз 18 ёшга тўлгунча қопланадиган алимент миқдоридан катта ёки тенг бўлса, алимент ўрнига уйни олишингиз мумкин. Каналимизга уланинг                 👇👇 👉 ‌‌
Show more ...
Юрист Хизмати Қабул
Ўзингизни қийнаётган барча саҳаларга оид муаммоли саволларингизни юборишингиз мумкин.
39 386
144
Бухгалтер ой ўртасида аванс тўламай баҳона қилиб юрса, қандай чора кўриш мумкин? 4466-савол: Assalamu alekum! Men Tuman Tibbiyot Birlashmasida navbatchi shifokor bo'lib ishlayman. Savolim: Oy o'rtasidagi avans to'lovi to'lash tartibi qanday bo'ladi? Men har safar avans tashlashlari uchun starshi bilan bugalterga iltimos qilishim kerak bugalter kech qoldiz avvalroq eslatishiz kerak edi yoki esimdan chiqibdi deydi bir tashlasa 2 3 oy yana boxona qilib tashlamaydi. Hozir 2 oydan buyon bo'ldi 100% tashlayman deb avans uchun moliya pul ajratmadi deyapti. Lekin shu tumandagi boshqa poleklinika xodimlariga 100% tashlagan. Shunday bo'lishi mumkinmi? Meni oyni o'rtasi va oxirida to'lovlarim bor, avansni regulyar xar oy tashlab borishi uchun nima qilishim kerak javob uchun oldindan raxmat! ЮРИСТ ЖАВОБИ: ❗️Аванс сизнинг талабингиз бўлганда тўланадиган ва бундай талаб қўймаганингизда тўланмайдиган тўлов эмас. Аксинча бунинг акси бўлиши керак. Қаранг! Меҳнат кодексининг 253-моддаси биринчи қисмида ходимларга иш ҳақи тўлаш муддатлари жамоа шартномасида ёки ички ҳужжатда, улар мавжуд бўлмаганда эса меҳнат шартномасида шарт қилиб кўрсатилган муддатларда белгиланади ва ҳар ярим ойда бир мартадан кам бўлиши мумкин эмаслиги белгиланмоқда. ☝️Бу қоиданинг ўзи иш ҳақи 1 ойда 2 қисмга бўлиб тўланишини ифода этмоқда. Қоидага кўра иш ҳақи ҳар ярим ойда (қоида тариқасида ҳар 15 кунда) 1 марта тўланиши шарт. Ушбу норманинг давомида айнан биз айтган тушунтириш берилмоқда. Яъни ходимларга ойлик иш ҳақи, қоида тариқасида, ўн олти кундан кўп бўлмаган танаффус билан икки қисмга бўлинган ҳолда (бўнакда ва қолган қисми миқдорида) тўланади. ❗️Бу ҳолатда сиз туман адлия бўлимига бухгалтернинг ҳаракатлари устидан шикоят қилишингиз мумкин. Каналимизга уланинг                 👇👇 👉 ‌‌
Show more ...
42 209
216
⚡️2024/2025 ўқув йилидан Ихтисослаштирилган таълим муассасалари агентлиги тизимидаги ижод ва ихтисослаштирилган мактабларда математика, физика, кимё, биология, табиий фанлар (science), информатика ва ахборот технологиялари фанларидан дарсларни чет тилида олиб борган ўқитувчиларга тариф ставкасига нисбатан 100 фоиз миқдорида ҳар ойлик қўшимча устама Президент Фармони. Каналимизга уланинг                 👇👇 👉 ‌‌
47 695
263
⚡️Президент мактабларининг маҳаллий ўқитувчиларига уларнинг тариф ставкасига нисбатан 350 фоиз миқдорида ҳар ойлик қўшимча устама тўлаш тартиби жорий этилади — Президент Фармони. Бунда, устама педагоглар томонидан қуйидаги шартларнинг барчаси бажарилган тақдирда : — Кембридж (Cambridge Assessment International Education) халқаро таълим дастурлари асосида дарс бериладиган фанларни ўқитиш бўйича 3 та (Introductory, Extension, Marking) курсни муваффақиятли тугатиш ва бу ҳақида тегишли сертификат олиш; — Президент мактабларида камида 1 йиллик иш стажига ва чет тилини билиш бўйича камида С1 даражадаги ёки унга тенглаштирилган мос даражадаги халқаро сертификатга эга бўлиш. Каналимизга уланинг                 👇👇 👉 ‌‌
Show more ...
44 185
188
Отангни машинаси бор деб бола пули беришни рад қилишлари тўғрими? 4465-савол: Assalomu aleykum mening 2 nafar voyaga yetmagan farzandim bor turmush o‘rtog‘im 4 yil avval avto halokat tufayli vafot etgan. Men 404000 so‘m bolalar nafaqasi olar edim.  Uning vaqti kelib tugadi shu sababli boshqatan ariza topshirish uchun mahallaga chiqsam ular senga chiqmaydi chunki dadangni mashinasi bor dedi.  Men o‘zim maktabda ishlayman oyligim 1200000 so‘m. Xozir qizlik uyimda yashayman meni nomimda na uy na yer bor. Lekin mahalladagilar sen dadangni mashinasini xaydading deb ro‘kach qilishyapti shu to‘g‘rimi oldindan raxmat. ЮРИСТ ЖАВОБИ: Вазирлар Маҳкамасининг 2021-йил 21-октабрдаги 654-сон қарорига 1-илова билан тасдиқланган “Кам таъминланган оилаларни “Ижтимоий ҳимоя ягона реестри” ахборот тизими орқали аниқлаш, уларга кам таъминланган оилалар болалари учун нафақа ва моддий ёрдам тайинлаш ва тўлаш тартиби тўғрисида”ги низомга кўра оила: 👉 2022-йилдан бошлаб — оиланинг ҳар бир аъзосига жами бир ойлик ўртача даромад минимал истеъмол харажатлари миқдоридан ошмаганда кам таъминланган деб эътироф этилади. Юқоридаги қоиданинг мазмунига кўра оилангизнинг даромади ҳозир ҳар бир оила аъзосига ҳисоблаганда 621 минг сўмдан кам бўлса, унда сиз кам таъминланган деб топилишингиз мумкин. Бунда ушбу низом 22-бандига кўра оиланинг бир аъзосига тўғри келадиган бир ойлик ўртача даромад болалар нафақаси ёки моддий ёрдам олиш учун мурожаат қилинган ойдан олдинги ойгача бўлган охирги уч ойи учун ҳисоблаб чиқилган барча оила аъзоларининг даромади суммасини оила таркибига киритиладиган оила аъзолари сонига ва учга бўлиш орқали аниқланади. Қаранг! Мазкур низомнинг 20-бандига кўра ариза берувчи оиласининг ҳар бир аъзосига тўғри келадиган бир ойлик ўртача жами даромад ушбу Низомнинг 7-бандида белгиланган мезондан ошганда болалар нафақаси ёки моддий ёрдам тайинлаш рад этилади. Шунингдек, мазкур бандга кўра ариза берувчи ёки унинг оила аъзолари фойдаланувида бўлган ёхуд ишончномага асосан ишлаб чиқарилган муддати 12 (оила аъзоси I ёки II гуруҳ ва 18 ёшгача болаликдан ногирон бўлса 7) йилдан ошмаган, техник ҳолати соз бўлган енгил автотранспорт воситаси бўлганда ҳам ариза рад қилинади. Энди қаранг! ушбу нормада оила аъзолари жумласи қўлланилган. Мазкур низом 22-бандига кўра болалар нафақаси ёки моддий ёрдам олиш учун оила таркибига: 👉 фуқаролик ҳолати далолатномаларини ёзиш органларида никоҳи қайд этилган эр-хотин (ёки уларнинг бири) ва улар билан бирга яшайдиган ва уларнинг қарамоғида бўлган болалар (шу жумладан, васийликка (ҳомийликка) олинган болалар); 👉 шунингдек, ота-оналари билан бирга яшайдиган ва ўз оиласига эга бўлмаган 18 ёшдан катта болалар (шу жумладан, васийликка (ҳомийликка) олинган болалар), ёлғиз оналар (оталар) киритилади. ❗️Шунга кўра сиздаги вазиятда ота-онангиз, сизнинг оила аъзойингиз таркибига кирмайди деб ҳисоблаймиз. Бу вазиятда туман адлия бўлимига мурожаат қилишингиз лозим. Каналимизга уланинг                 👇👇 👉 ‌‌
Show more ...
50 000
256
❗️Ички ишлар вазири 31 март - якшанба куни, 1 апрелдан автомобилларни давлат рўйхатидан ўтказиш 68 бараварга – 34 минг сўмдан 2,32 млн сўмгача ошириш тўғрисидаги буйруқ Жамоатчилик муҳокамасига қўйилмаган, нархи асосланмаган бу буйруқ бўйича очиқ саволлар ва аҳолининг фикри ҳақида қуйидаги видеода Каналимизга уланинг                 👇👇 👉 ‌‌

68_бараварга_оширилган_хизмат_бизни_ўйлашмайдими_фикримиз_қи.mp4

46 859
267
ТАЪТИЛ ПУЛИ КУЙИБ КЕТАДИМИ? 4464-савол: Ассалому алейкум! Мен мактабда қоровул булиб ишлайман. Менинг навбатдаги мехнат таътилига чиқиш вақтим 1 мартдан деб айтган эди. Мен 5 март куни мехнат таътилига чиқиш учун ариза ёзишимни айтсам, менга, мехнат таътилим вахти утиб, куйиб кетганини айтди. Шу туғрими? ЮРИСТ ЖАВОБИ:Таътил пули куйиб кетмайди. ❗️Таътил олиш ходимнинг ҳуқуқи ва иш берувчининг мажбурияти. Бироқ ходим таътил муддати келганда уни олиши шарт. Ҳар йилги меҳнат таътили иш берувчи ва ходим учун мажбурий бўлган таътиллар жадвали билан белгиланадиган ҳар йилги меҳнат таътилларини бериш навбатига мувофиқ берилади (МК 228-м). Шунга кўра ташкилотда ҳар йиллик меҳнат таътиллари жадвали кейинги йил бошланишидан олдин, қоида тариқасида декабр ойида тузилади. Шу жадвалга мувофиқ ходимга албатта таътил берилади. 💁‍♂ Шуни ҳам айтиш керакки, Меҳнат кодекси 234-моддасига асосан меҳнат шартномаси бекор қилинганда ходимга барча фойдаланилмаган ҳар йилги асосий ва қўшимча меҳнат таътиллари учун пулли компенсация тўланади. Бунда ходим билан меҳнат шартномаси бекор қилинганда жорий иш йилида фойдаланилмаган ҳар йилги меҳнат таътили учун ходимга тегишли бўлган компенсациянинг миқдори, шунингдек жорий иш йилида ишлаб берилмаган таътил кунлари учун ушлаб қолинадиган ҳақ миқдори ходимнинг охирги иш йилида ишлаб берган вақтига мутаносиб равишда ҳисоблаб чиқарилади (МК 223-м). Шунга кўра ишдан бўшаганингизда Меҳнат кодексининг юқоридаги ҳамда 172-моддасига мувофиқ сизга фойдаланилмаган меҳнат таътили учун компенсация берилади. ✅ Агар сиз таътил олмасангиз, ҳам таътилингиз куйиб кетмайди. Ишдан бўшаган вақтингизда таътил учун компенсация олишингиз мумкин. Каналимизга уланинг                 👇👇 👉 ‌‌
Show more ...
41 214
224
Ўқитувчи (PhD) бўлса, унга қўшимча устама тўланадими? Дарс тақсимотида бирор устунликка эгами? 4463-савол: Rus tili  o’qituvchisi (Phd )fan nomzodi, maktabda faoliyat olib borsa unga ustama to’lanadimi, dars taqsimotida qanday ustunlik mavjud toifali o’qituvchilarga nisbatan. ЮРИСТ ЖАВОБИ: ✅ Бундай ҳолатда сизга махсус устама мавжуд. Вазирлар Маҳкамасининг 2019-йил 24-декабрдаги 1030-сон қарорига 1-иловада тасдиқланган “Илм-фан ва таълим соҳасидаги давлат ташкилотларида илмий, илмий-педагогик ва меҳнат фаолияти билан шуғулланувчи илмий даражага эга ходимларга қўшимча ҳақ тўлаш тартиби тўғрисида”ги низом 14-бандига мувофиқ лавозим маошига ҳар ойлик қўшимча ҳақ ходимнинг фаолияти натижалари асосида қуйидагича ўрнатилиши мумкин: а) ёшларни тарбиялашга оид ҳар бир ишлаб чиқилган ва амалиётга жорий қилинган илмий-методик қўлланма, илмий-таҳлилий услуб (ёндашув)лар учун лавозим маошига нисбатан 30 фоизгача; б) таълим ва тарбия соҳасида тегишли ташкилот билан боғлиқ илмий мақола (монография) нашр қилганлиги учун: 👉 журналнинг даражаси ва муҳимлигидан келиб чиққан ҳолда миллий илмий журналлардаги нашр учун лавозим маошига нисбатан 15 фоизгача; 👉 халқаро журналларда эълон қилинган ҳар бир мақола (монография) учун муаллифларга лавозим маошига нисбатан 30 фоизгача белгиланади. Мазкур банднинг барча кичик бандлари асосида ҳар ойлик қўшимча ҳақ лавозим маошига нисбатан бир ойда фалсафа доктори (PhD)га 30 фоиз ва фан доктори (DSc)га 60 фоиздан ошмаслиги лозим. ☝️Шу қоидага кўра сизга юқоридаги устама тўланиши мумкин. ❌ Дарс тақсимотида бирор имтиёз мавжуд эмас деб ҳисоблаймиз. Каналимизга уланинг                 👇👇 👉 ‌‌
Show more ...
45 155
188
❗️"Метан" нархини асоссиз оширган заправкаларга чора кўрилади Бугун ижтимоий тармоқларда “метан заправка”ларда газ нархи 3 800 сўмга кўтарилгани ҳақида хабарлар эълон қилинди. Ушбу ҳолат Энергетика вазирлиги ҳузуридаги “Ўзенергоинспексия” томонидан зудлик билан “Ўзенергоинспексия”нинг ҳудудий бошқармалари томонидан "метан" нархини 3800 сўмга оширган заправкалар зудлик билан ўрганилди ва бартараф этилди. Эслатиб ўтамиз, Вазирлар Маҳкамасининг 2024 йил 30 мартдаги 173-сонли қарори билан автомобилларга газ тўлдириш компрессор шохобчаларида табиий газни сотишда унинг энг баланд чакана нархи 2024 йил 1 июнгача бир куб метр учун 3 350 сўм қилиб белгиланган. Агар АГТКШларда табиий газ асоссиз равишда қиммат нархларда сотилаётганига гувоҳ бўлсангиз, қуйидаги манзилларга мурожаат қилишларингизни сўраймиз: 👉 Бош прокуратуранинг telegram боти ва ишонч телефонига NaN; 👉 “Ўзенергоинспексия”нинг telegram боти ва ишонч телефонига NaN; 👉 Рақобатни ривожлантириш ва истеъмолчилар ҳуқуқларини ҳимоя қилиш қўмитасининг ишонч телефонига NaN. Каналимизга уланинг                 👇👇 👉 ‌‌
Show more ...
59 725
318
Тиббиёт ҳамшираларига неча ставкагача ишлашга рухсат берилган? 4462-савол: Ассалому алекум. Мен тиббиет хамширасиман. Поликлиникада 1.5Ставка сурадим Сенга мумкинмас дейшди. Нима маслахат берасиз. Тиббиет хамшираси 2 ставка ишлатса хаклими  2ставкага буйрук борми. Мальумот берсангиз ЮРИСТ ЖАВОБИ: ❗️Врачларгина 2 ставка ишлаши мумкин. Вазирлар Маҳкамасининг 297-сон қарори билан тасдиқланган низом 7-бандига кўра ўриндошлик асосида ишлашда ўриндошлик асосида ишлашнинг давомийлиги ходимларнинг ушбу тоифаси учун белгиланган иш вақти нормасининг ярмидан ортиқ бўлиши мумкин эмас. Бундан фақат соғлиқни сақлаш тизими врач ходимлари мустасно. ✅ Шунга кўра сиз асосий ва ўриндошликда 1.5 ставкагача ишлашингиз мумкин. Меҳнат кодексининг 432-моддасига кўра Ўриндошлик асосида ишлаш ходимнинг ўзининг асосий ишидан ташқари алоҳида меҳнат шартномаси шартлари бўйича асосий ишидан бўш вақтида бошқа мунтазам ҳақ тўланадиган ишни бажаришидир. Ўриндошлик асосидаги иш ходим томонидан ўзининг асосий иш жойида (ички ўриндошлик) ёки бошқа иш берувчида (ташқи ўриндошлик) бажарилиши мумкин. ☝️Юқоридаги икки қоидага мувофиқ сизнинг меҳнат дафтарчангиз юритиладиган ҳамда маълум лавозимда ишлайдиган жой бу сизнинг асосий иш жойингиз саналади. Сиз ушбу жойдан ташқари қаерда ишлашингиздан қатъий назар бу ўриндошлик ҳисобланади. ✅ Шунга кўра, сиз асосийда бир ставка, ўриндошликда 0.5, жами 1.5 ставкагача ишлашингиз мумкин. Каналимизга уланинг                 👇👇 👉 ‌‌
Show more ...
42 799
240
НОТАРИУСНИНГ ТАЛАБИ ТЎҒРИМИ? 4461-савол: Assalomu alaykum meni dadam 2023-yil sentabr oyida olamdan o’tgan. Dadamning nomidagi uyni onamning nomiga otkazmoqchi bolib natariusga borgandik natarius 1960-yillar atrofida otib ketgan katta otamning katta onamning (dadamning ota-onasi) olim haqidagi guvohnomalarni va biz aka-uka, opa-singillarimizning rozilik xati kerak dedi. Endi sizlarga savolim bu vaziyatda nma qilishimiz kerak aka-ukalarimiz rassiyada opa-singillarimiz boshqa viloyatlarga turmush qilib ketgan, dadamning nomidan onamning nomiga uyni otkazish uchun shuncha hujjat toplanish shartmi ЮРИСТ ЖАВОБИ:Нотаируснинг талаби тўғри. Чунки мерос иши очилганда меросхўларнинг ҳар бир бўлиши шарт. Лекин Фуқаролик кодексининг 1151-моддаси иккинчи қисмига кўра агар турган жойи аниқланиб, мерос олишга чақирилган-у, лекин ҳозир бўлмаган меросхўр меросдан воз кечмаган бўлса, қолган меросхўрлар меросни тақсимлаш ниятлари тўғрисида уни хабардор қилишлари шарт. Агар ҳозир бўлмаган меросхўр хабар қилинган пайтдан эътиборан уч ой ичида қолган меросхўрларни меросни тақсимлаш ҳақидаги келишувда иштирок этиш истаги тўғрисида хабардор қилмаса, қолган меросхўрлар ҳозир бўлмаган меросхўрга тегишли бўлган улушни ажратиб, ўзаро келишувга кўра мерос тақсимотини амалга оширишга ҳақлидирлар. ☝️Мана шу қоидага кўра сиз нотариусдан ҳозир бўлмаган меросхўрларнинг улушларини ажратган ҳолда мерос гувоҳномаси расмийлаштиришни сўрашингиз мумкин. ➡️ Бобо ҳамда бувингизнинг маълумотларга келсак, Вазирлар Маҳкамасининг 2020 йил 18 ноябрдаги 726-сон қарорига 5-иловада тасдиқланган Нотариуслар томонидан меросга бўлган ҳуқуқ тўғрисида гувоҳномани беришнинг маъмурий Регламенти 29-бандига кўра меросхўрларнинг туғилганлик ҳақидаги гувоҳномалари ва никоҳ тузилганлиги ҳақидаги гувоҳномалари (Ф.И.О. ўзгарган тақдирда) ёки далолатнома ёзувларидан кўчирмалар нотариусга тақдим этилиши керак. Лекин меросхўрлар ушбу  маълумотларни тақдим этиш имкони бўлмаган тақдирда, ушбу маълумотлар нотариал идоралар томонидан мустақил равишда, «ФҲДЁнинг ягона электрон архиви» ахборот тизими орқали олиниши мумкин. Шунга кўра нотариус бобонгиз ҳамда бувингизнинг маълумотларини базадан ўзи олиши мумкин. 💁‍♂ Шуни айтиш керакки, чет элдаги меросхўрлар меросдан воз кечиш ҳақида ўзи яшаётган давлатдаги нотариал идорада воз кечиш ҳақида ариза тақдим этса, сиз ушбу аризаларни нотариусга тақдим этишингиз мумкин. Каналимизга уланинг                 👇👇 👉 ‌‌
Show more ...
Юрист Хизмати Қабул
Ўзингизни қийнаётган барча саҳаларга оид муаммоли саволларингизни юборишингиз мумкин.
41 799
105
❗️1 апрелдан нималар ўзгаради? ➖ Тадбиркорларга субсидия беришнинг янги тартиби жорий этилади; ➖ Тўсиқсиз меҳмонхоналарга – ҳар бир хона учун 1 млн сўмдан 2 млн сўмгача субсидия берилади; ➖ Давлат харидларида қатнашувчи маҳаллий ишлаб чиқарувчиларга преференциялар берилади; ➖ Йирик солиқ тўловчиларга электрон юк хатлари жорий этилмоқда; ➖ Чиқинди учун қарздорлик мавжудлигида кўчмас мулк ва автотранспортни нотариал тартибда расмийлаштириш рад этилади; ➖ “Инсон” ижтимоий хизматлар марказлари пуллик асосда хам хизмат кўрсатишади; ➖ ҚҚС бўйича айрим имтиёзлар бекор қилинмоқда; ➖ Моддий ёрдам тайинлаш  онлайн амалга оширилади; ➖ Архитекторларни сертификатлаштириш тизими жорий этилади; ➖ Банклар учун устав капиталининг энг кам миқдори белгиланмоқда; ➖ IT-паркнинг айрим резидентлари ортиқча қоғозбозликдан озод этилади; ➖ Фарзанд туғилганда тўланадиган нафақа тайинлаш тартиби ўзгармоқда; ➖ Янги маданий лойиҳалар амалга оширилади; 👉 Каналимизга уланинг                 👇👇 👉 ‌‌
Show more ...
1 апрелдан нималар ўзгаради
Norma.uz портали 2024 йил 1 апрелдан кучга кирадиган миллий қонунчиликдаги асосий ўзгартиришлар шарҳини тақдим этади.
49 617
218
❗️Метан газнинг энг юқори нархи 3350 сўм этиб белгиланди Ўзбекистон Республикаси Вазирлар Маҳкамасининг 2024 йил 30 март куни қабул қилинган 173-сонли эртага, 1 апрелдан эътиборан «метан» газнинг энг баланд чакана нархи 2024 йил 1 июнгача бир куб метр учун 3 350 сўм қилиб белгиланди. Белгиланган муддат яна икки ой давомида жузъий ўзгартишлар билан амал қилади (аввалги нарх 3 250 сўм эди). Каналимизга уланинг                 👇👇 👉 ‌‌
55 166
320
Куёвим бошқасига уйланволди, қизим ажрашса бўладими? 4460-савол: Ассалому алайкум! қизим 2023 йил турмушга чиқди лекин 2024 йил 21 февралда қайнонаси кетириб куйиб кетди чунки ўғлини қилган зуғими уриб хўрлаганлиги ва кет деб кучага судраб ташаганлиги учун қайнонаси уйизга туринг деб ёлғондан куйиб кетди бунга сабаб ўртада бошқа аёл бўлган мақсади уни олиб келиб яшаш бўлган куёвим қонунан қизим билан яшасада бошқасини олиб келган қизим у хонадонда жуда азият чеккан шунга қарамай чидаб хайдаманг деб ёлворган лекин афсуски хеч ким инобатга омаган қизим ўз кузлари билан 2 чи аёлини кургандан кейин мажбур бўлдим ажратишга лекин ажримни эри берсин деб талаб қидим лекин натижа булмади илтимос маслахат беринг ЮРИСТ ЖАВОБИ: Ажримга ким ариза беришини аҳамияти йўқ, чунки суд аризани ким берганига эмас, оилани асраб қолиш мумкин ёки йўқлигини текширади. Ажрашиш жараёни жуда қийин ҳисобланади. Оила кодекси 37-моддаси иккинчи қисмига мувофиқ никоҳ эр-хотиндан бири ёки ҳар иккаласининг аризасига мувофиқ никоҳдан ажратиш йўли билан, шунингдек суд томонидан муомалага лаёқатсиз деб топилган эр ёки хотиннинг васийси берган аризага мувофиқ тугатилиши мумкин. Энди қаранг! Оила кодекси 40-моддаси ҳамда Олий суди Пленумининг 2011 йил 20 июлдаги 06-сонли «Судлар томонидан никоҳдан ажратишга оид ишлар бўйича қонунчиликни қўллаш амалиёти тўғрисида»ги қарорига мувофиқ эр-хотинга ярашиш учун олти ойгача муҳлат тайинлашга ҳақли. Бунда шуни эътиборга олиш лозимки, Оила кодексининг 218-моддаси талабларини инобатга олган ҳолда ярашиш учун бериладиган муҳлат уч ойдан кам бўлиши самарасиз ҳисобланади. Шунга кўра сизлардаги вазиятда қизингиз ва куёвингизга ярашиш учун 3 ойдан 6 ойгача муҳмат бериши мумкин.  Бу муддат тугагандан сўнг суд ишни кўриб чиқиши лозим. Бунда ишни мазмунан кўриб ҳал қилиши, яъни никоҳни бекор қилиши ёки аризани рад қилиши мумкин. Оила кодекси 41-моддаси ҳамда Олий суди Пленумининг 2011 йил 20 июлдаги 06-сонли қарорининг 16-бандларига таянган ҳолда никоҳдан ажратиш тўғрисидаги талаб, фақат, эр-хотин бундан буён бирга ҳаёт кечиришларининг ва оила батамом бузилганлиги сабабли уни сақлаб қолишнинг иложи йўқлиги аниқланган ҳолдагина қаноатлантирилиши лозим. Шундай экан, агар суд оиланинг кейинчалик бирга яшаб кетиш имкони йўқ деб топсагина, никоҳни бекор қилиши мумкин. Сиздаги ҳолатда бир томон қарши бўлган тақдирда ҳам суд никоҳни сақлаб қолиш учун имкон йўқ деб топса, никоҳ бекор бўлади. Ёки ҳар икки томон никоҳдан ажратишни хоҳлаган тақдирда ҳам суд оилани сақлаб қолиш учун имкон бор деб топса, даъвони рад қилади. 💁‍♂ Шуни ҳам айтиш керакки, бу вазиятда қизингиз ва куёвингизнинг янги хотини 1 уйда яшаса, куёвингизга жавобгарлик бор. Жиноят кодекси 126-моддасига кўра кўп хотинли бўлиш, яъни умумий рўзғор асосида икки ёки ундан ортиқ хотин билан эр-хотин бўлиб яшаш — базавий ҳисоблаш миқдорининг 50 бараваридан 100 бараваригача миқдорда жарима ёки 3 йилгача ахлоқ тузатиш ишлари ёки бир йилдан уч йилгача озодликни чеклаш ёхуд уч йилгача озодликдан маҳрум қилиш билан жазоланади. ❗️Шунга кўра агар куёвингиз бир уйда икки хотин билан яшасагина унга юқоридаги модда билан жиноий иш очилиши мумкин. Каналимизга уланинг                 👇👇 👉 ‌‌
Show more ...
42 814
164
Тиббиётда ҳам жуда катта штат қисқартиришлари бўлиши мумкин... Соғлиқни сақлаш ташкилотларида қисқартирилиши мумкин бўлган штатлар ҳақидаги маълумотлар . Каналимизга уланинг                 👇👇 👉 ‌‌
50 324
544
❗️Ота-оналар томонидан педагогларга тахдид, туҳмат қилинса, чора борми? 4459-савол: Assalomu alekum ota onalar tamonidan pedagoglarga taqdid tuxmat qilishga chora kuriladimi? ЮРИСТ ЖАВОБИ: Ўзбекистон Республикаси қонунчилигига кўра ҳар қандай ҳолатда ҳам шахсни ҳақорат қилиш, унга туҳмат қилиш тақиқланган. ❕Шуни айтиш керакки янги таҳрирдаги Конситуциямизнинг 52-моддаси иккинчи қисмида давлат ўқитувчиларнинг шаъни ва қадр-қимматини ҳимоя қилиш, уларнинг ижтимоий ва моддий фаровонлиги, касбий жиҳатдан ўсиши тўғрисида ғамхўрлик қилиши мустаҳкамлаб қўйилган. ❌ Шунинг учун ўқитувчиларни ҳақорат қилиш, уларнинг устидан кулиш ёки уларга тухмат қилиш кабилар мумкин бўлмайди. Аксинча мазкур қилмишларни содир этган шахсларга нисбатан маъмурий ва жиноий жавобгарлик чоралари белгиланган. Хусусан, Маъмурий жавобгарлик тўғрисидаги кодекс 41-моддасига мувофиқ ҳақорат қилиш, яъни шахснинг шаъни ва қадр-қимматини қасддан камситиш — базавий ҳисоблаш миқдорининг 20 бараваридан 40 бараваригача миқдорда жарима солишга сабаб бўлади. ❗️Энди қаранг! агар юқоридаги қилмишлар маъмурий жазо қўлланилгандан кейин 1 йил давомида қайта содир этилса унда бу қилмишларга нисбатан жиноий жавобгар чораси қўлланади. (Жиноят кодекси 139-моддаси туҳмат, 140-моддаси ҳақорат) Каналимизга уланинг                 👇👇 👉 ‌‌
Show more ...
56 288
667
❗️Штатларни қисқартириш ўрнига, штатларни кўпайтириш таклифини киритиш керакмасми Штатларни кўпайтириш ўрнига, фақат қисқартиришлар қилинмоқда. У ҳолда шунча етишаётган кадрлар қандай иш топа олади? Қисқаришга тушган ходимлар қандай иш топади? Ёки чет элга ишлашга кетадими? Тангани икки томонини ҳам кўриб иш қилиш керак. 👉
41 034
242
Иқтисодиёт ва молия вазирлиги  ҳамма соҳада қисқартириш таклифини киритяпти Унга кўра: — Туман (шаҳар) адлия бўлимлари ва Халқ қабулхоналари тугатилиши мумкин. — Ҳудудий Ғазначилик хизмати бошқармалари ҳамда Бюджетдан ташқари Пенсия бошқармалари ҳам тугатилади.Умумтаьлим, мусиқа мактаблари, ўрта махсус ва профессионал таьлим ташкилотларидаги бир қанча лавозимлар ҳам . — Соғлиқни сақлаш ташкилотларида ҳам штатлар қисқартирилади. Каналимизга уланинг                 👇👇 👉 ‌‌
Show more ...
52 233
1 299
32 йил стажи бор педагог аттестацияга жалб этиладими? 4458-савол: Assalomu aleykum maktab faoliyatida 32 yil ishlagan óqituvchi toifasi mutaxasis bólsa attestatsiyaga jalb qilinadimi? Yoki yóqmi ЮРИСТ ЖАВОБИ: Вазирлар Маҳкамасининг 2021 йил 17 сентябрдаги 572-сон қарорига 1-илова билан тасдиқланган “Мактабгача, умумий ўрта, ўрта махсус, профессионал ва мактабдан ташқари таълим ташкилотлари педагог кадрларини аттестациядан ўтказиш тартиби тўғрисида”ги низом 4-бандига кўра мажбурий аттестация 5 йилда ўтказилади. ❗️Саволда 2021 йилда борганингизни ёзгансиз, шундай экан сизни бу йил мажбурий аттестацияга жалб қилиш мумкин эмас. Иккинчидан, низом 6-бандига асосан: 👉 педагогик фаолият бўйича 25 йил ва ундан кўп иш стажига эга бўлган педагог ходимлар; 👉 ёшга доир пенсияга чиқиши учун 5 йилдан кўп бўлмаган вақт қолган педагог ходимлар навбатдаги мажбурий аттестацияга жалб этилмайди. Бунда 8-банди Вазирлар Маҳкамасининг 2023 йил 29 декабрдаги 692-сонли қарорига асосан «е» кичик банд билан тўлдирилган, унга кўра педагогик фаолият бўйича 25 йил ва ундан кўп иш стажига эга бўлган ҳамда ёшга доир пенсияга чиқиши учун 5 йилдан кўп бўлмаган вақт қолган педагог кадрларга — уларнинг амалдаги малака тоифаси (лавозими) берилади. ❗️Шундай экан, сизда педагогик соҳа бўйича 32 йиллик иш стажи мавжуд бўлгани туфайли сиз мажбурий аттестацияга жалб этилмайсиз ва сизнинг тоифангиз сақланиши керак деб ҳисоблаймиз. Яъни бунда ўзингиз хоҳласангиз, мутахассис тоифада қолишингиз мумкин. Каналимизга уланинг                 👇👇 👉 ‌‌
Show more ...
42 112
358
⚡️Умумий ўрта таълим муассасаларида якуний давлат аттестацияси ўтказиш тартиби тасдиқланди Мактабгача ва мактаб таълими вазирининг 2024 йил 20 мартдаги тегишли буйруғи билан: ❗️9-синф ўқувчиларининг якуний давлат аттестацияси: ➖Она тили ва адабиёти фанидан — ёзма; ➖Давлат тили (таълим тили ўзбек тили бўлмаган синфлар учун)дан — ёзма; ➖Математика фанидан — ёзма; ➖Ўзбекистон тарихи фанидан — ёзма/оғзаки; ➖Жисмоний тарбия фанидан — амалий шаклда ўтказилади. ❗️11-синф ўқувчиларининг якуний давлат аттестацияси: ➖Она тили ва адабиёти фанидан — ёзма (иншо); ➖Математика фанидан — ёзма; ➖Давлат тили (таълим ўзбек тилида олиб борилмайдиган умумий ўрта таълим муассасалари учун, таълим муассасаси танлови асосида)дан — ёзма ва оғзаки; ➖Чет тили фанидан  — ёзма/оғзаки; ➖Ўзбекистон тарихи фанидан — ёзма/оғзаки; ➖(Ўқувчининг танлови бўйича) Кимё, Физика, Давлат ҳуқуқ асослари, География фанларидан — ёзма шаклда ўтказилади. Маълумот учун, 2025 йилидан бошлаб таълим ўзбек тилида олиб борилмайдиган умумий ўрта таълим муассасаларининг 11-синф битирувчилари учун Давлат тилидан якуний назорат имтиҳонларини топшириш мажбурий бўлиши . Каналимизга уланинг                 👇👇 👉 ‌‌
Show more ...
65 636
770
Лизинга автомашина олмоқчи эдим, алданиб қолмаслик учун нималарга аҳамият беришим керак? 4457-савол: Лизинга автомашина олмокчи эдим, алданиб колмаслик учун нималарга ахамият беришим керак булади. ЮРИСТ ЖАВОБИ: ❗️Лизингга енгил автротранспорт воситаси олмасликни тавсия қиламиз. Негаки ҳозирда кўплаб лизинг фирмалари тузаётган лизинг аслида лизинг эмас. Чунки Лизинг тўғрисидаги қонун 3-моддасига кўра тадбиркорлик фаолияти учун фойдаланиладиган истеъмол қилинмайдиган ҳар қандай ашёлар, шу жумладан корхоналар, мулкий комплекслар, бинолар, иншоотлар, ускуналар, транспорт воситалари ҳамда бошқа кўчар ва кўчмас мулк лизинг обектлари бўлиши мумкин. 💁‍♂ Шунга кўра аслида лизинг фақат тадбиркорлик фаолиятида фойдаланиш учун бериладиган транспорт воситалари учун берилади. 🙅‍♂ Шахсий ёки розғордаги эҳтиёжлар учун лизинг берилмайди. ✅ Агар лизинг олмоқчи бўлсангиз, сизга расмий дистрибютерлар орқали лизинг олишни тавсия қиламиз. Каналимизга уланинг                 👇👇 👉 ‌‌
Show more ...
43 907
74
⚡️Ўқувчиларнинг якуний назорат имтиҳонлари бўйича ММТВ нинг 87-буйруғи ММТВ нинг 20.03.2024 йил.Ўқувчиларнинг  якуний давлат  аттестациялари тўғрисидаги 87-  иловалари билан Каналимизга уланинг                 👇👇 👉 ‌‌

Якуний_имтихон_2024_йил_иловалари_билан.pdf

65 032
1 036
Раҳбар ходимлар бола парваришлаш таътилига чиқиши мумкинми? 4456-савол: Мактаб раҳбарлари  бола парвариш (навирасини парвариш ) таьтилига чикиши мумкинми? ЮРИСТ ЖАВОБИ:Ҳа албатта мумкин. Амалдаги меҳнат қонунчилиги ходимларга бериладиган бола парваришлаш таътилига оид ҳуқуқларда уларнинг тоифасини ажратмайди. Меҳнат кодексининг 405-моддасига асосан болани парваришлаш таътилларидан боланинг отаси, бувиси, буваси ёки болани ҳақиқатда парваришлаётган бошқа қариндошлари ҳам, шунингдек васийси ҳам тўлиқ ёки қисмларга бўлиб фойдаланиши мумкин. ❗️Шунингдек, ушбу моддага кўра болани парваришлаш таътиллари даврида ходимнинг иш жойи (лавозими) сақланади. Бу таътилларнинг кўпи билан олти йили меҳнат стажига қўшилади, шу жумладан мутахассислиги бўйича иш стажига ҳам қўшиб ҳисобланади. Бунда иш берувчи ушбу ходим ўрнига бошқа ходимни ишга қабул қилиш мумкин. Бунинг ҳуқуқий асослари мавжуд. Меҳнат кодексининг 112-моддасига мувофиқ меҳнат тўғрисидаги қонунчиликка ва меҳнат ҳақидаги бошқа ҳуқуқий ҳужжатларга, меҳнат шартномасига мувофиқ иш жойи ўзида сақланиб қоладиган, йўқ бўлган ходимнинг мажбуриятларини бажариш вақти учун муддатли меҳнат шартномаси асосида янги ходимни ишга қабул қилиш мумкин бўлади. Ушбу кодекс 237-моддасида ҳомиладорлик ва туғиш таътиллари, бола парваришлаш таътили ижтимоий таътил эканлиги келтирилган. Энди қаранг! МК 239-моддасида барча ижтимоий таътиллар вақтида ходимнинг аввалги иш жойи (лавозими) сақланиб қолиши белгиланган. ☝️Мана шу қоидага кўра иш берувчи декретга кетган ходим ўрнига янги ходимни ишга олиши мумкин. Бунда иш берувчи муддатли меҳнат шартномаси асосида бошқа ходимни ишга олиши мумкин бўлади 💁‍♂ Шуни ҳам айтиш керакки, ушбу кодекс 158-моддаси учинчи қисмига мувофиқ иш жойи (лавозими) сақланаётган ходимнинг йўқ бўлган вақтига тузилган меҳнат шартномаси ушбу ходим ишга қайтган кундан эътиборан бекор қилинади. Бунда ҳеч қандай огоҳлантириш талаб этилмайди. Каналимизга уланинг                 👇👇 👉 ‌‌
Show more ...
43 505
174
Last updated: 11.07.23
Privacy Policy Telemetrio